Identify Assumptions: Steps to Question Underlying Beliefs in Decision-Making

Identify Assumptions

Assumptions are the foundation upon which we base our decisions, thoughts, and actions, often without conscious awareness. These implicit beliefs or suppositions have a substantial impact on how we interpret the world and act within it.

Identifying assumptions is thus a crucial skill that requires both attention and critical thinking, enabling individuals to dissect their reasoning processes and the premises upon which they rely.

A table with two people sitting across from each other, pointing at a diagram on a piece of paper. There are pens and notebooks on the table

For scholars and professionals alike, being able to distinguish between what is assumed and what is known is essential for robust analysis and innovation.

It often involves a diligent review of the current knowledge base, followed by a systematic approach to test and validate the underlying presumptions.

As such, this careful interrogation of assumptions can pave the way for more informed decisions and groundbreaking discoveries.

Key Takeaways

Understanding assumptions.

Assumptions are the unspoken bedrock of beliefs and propositions that shape the way individuals perceive the world and inform their decision-making process. These underpinnings, often accepted as truth, play a critical role in forming perspective and dictate the importance placed on certain topics or actions.

Types of Assumptions

Importance of identifying assumptions.

In research, for example, assumptions can significantly affect the design and interpretation of studies.

Challenging Assumptions

Assumptions in decision-making.

Assumptions play a critical role in decision-making, shaping planning and strategy, influencing leadership, and determining outcomes. They form the invisible bedrock on which the edifice of choices is built.

Role in Planning and Strategy

It’s vital to recognize that these are not certainties but educated guesses designed to provide a foundation for planning .

A technique often used is a SWOT analysis (Strengths, Weaknesses, Opportunities, Threats), requiring a clear separation of factual data from assumptions to guide strategic direction clearly.

Impact on Leadership and Outcomes

This acknowledgement can steer leaders away from biases and towards outcomes that are more aligned with the reality of their operating environment.

Assumptions Template for Effective Choices

Assumptions in research and innovation, influence on new business development.

New business development hinges on a series of critical assumptions about market needs, customer behavior, and the anticipated impact of new offerings.

Driving Forces Behind Creativity and Change

In the realm of creativity and change, assumptions play a dual role.

They are often the invisible driving forces that compel individuals to seek out novel solutions and foster an environment conducive to innovation .

Engineering Assumptions in Product Design

When applied to product design , engineering assumptions are foundational to the process of design thinking .

Testing and Validating Assumptions

Before launching a product or implementing a strategy, assumptions must be rigorously tested and validated. This ensures that decisions are made on the basis of sound data and realistic expectations, mitigating the risks inherent in untested beliefs.

Methods for Assessing Hypotheses

Assumptions can be categorized by their likelihood and impact, assisting teams in focusing on those that are critical to success.

Utilizing Feedback and Databases for Refinement

This can take the form of user testing, surveys, or prototype trials, where real user interactions provide concrete evidence to support or refute the initial hypotheses.

Both sources of feedback help refine the assumptions and guide product development or business strategy changes based on this empirical feedback.

Learning from Failure and Success

This is a fundamental component of assumption testing ; recognizing failure early allows for a less costly course correction.

Assumptions in Project Management

In project management, assumptions significantly influence the planning and execution phases. They guide decision-making and strategizing throughout the life cycle of a project.

Defining Objectives and Key Results

For instance, project managers may assume that their teams will have access to certain services or technology, setting the stage for the desired outcomes.

Dealing with Constraints and Project Progress

If managers assume that they have more budget flexibility than they actually do, it could lead to unrealistic planning and eventual barriers in project delivery.

Adapting to Variables and Conditions

Assumptions about external factors such as regulatory rules or economic fluctuations are necessary for developing contingency plans.

Frequently Asked Questions

What are common examples of assumptions made in psychological research.

In psychological research, assumptions often include beliefs about human nature, such as intrinsic motivation or the universality of emotional expressions. These underlying beliefs can shape both the design and interpretation of studies.

How do assumptions affect the outcomes of critical thinking processes?

What methods are employed to identify assumptions in empirical research.

Empirical research often employs strategies such as scrutinizing cause-and-effect relationships and questioning the validity of operational definitions to identify assumptions.

In the context of econometrics, how are identifying assumptions integral to model specification?

Why is it crucial to distinguish assumptions when conducting any form of empirical analysis.

It is crucial to distinguish assumptions in empirical analysis because they form the backbone of research frameworks.

What are the consequences of not accurately identifying assumptions in academic research?

This may ultimately diminish the study’s reliability and impact its success .

You may also like

A student’s guide to critical thinking, best movies for critical thinking: top picks to challenge your mind, critical thinking job interview questions, the basics of using critical thinking in business, download this free ebook.

  • Archives & Special Collections home
  • Art Library home
  • Ekstrom Library home
  • Kornhauser Health Sciences Library home
  • Law Library home
  • Music Library home
  • University of Louisville Hospital home
  • Interlibrary Loan
  • Off-Campus Login
  • Renew Books
  • Cardinal Card
  • My Print Center
  • Business Ops
  • Cards Career Connection

Search Site

Search catalog, critical thinking and academic research: assumptions.

  • Information
  • Point of View
  • Assumptions
  • Implications

Question Assumptions

An assumption is an unexamined belief: what we think without realizing we think it. Our inferences (also called conclusions) are often based on assumptions that we haven't thought about critically. A critical thinker, however, is attentive to these assumptions because they are sometimes incorrect or misguided. Just because we assume something is true doesn't mean it is.

Think carefully about your assumptions when finding and analyzing information but also think carefully about the assumptions of others. Whether you're looking at a website or a scholarly article, you should always consider the author's assumptions. Are the author's conclusions based on assumptions that she or he hasn't thought about logically?

Critical Questions

  • What am I taking for granted?
  • Am I assuming something I shouldn't?
  • How can I determine whether this assumption is accurate?
  • What is this author assuming?
  • How can I determine if this author's assumptions are accurate?

Consider the following situations, then respond to these questions:

  • Do you agree or disagree with the inference/conclusion? Why or why not?
  • What assumption(s) may have led to the inference/conclusion?
  • What are some alternative ways of thinking about this situation?

Situation #1

Bill needs six scholarly articles for his paper on the psychological effects of domestic violence. He searches Google for "psychological effects of domestic violence," looks through the first few hits, and finds six sources, including some articles on the websites of legitimate organizations. A few of these articles include bibliographies.

  • Bill's Inference/Conclusion: I'm going to stop researching because I have my six sources.

Situation #2

Christie is researching representations of gender in popular music. She decides to search Google and, within a few minutes, locates more sources that she could possibly incorporate into her final paper.

  • Christie's Inference/Conclusion: I can just use Google for my research.

Situation #3

Jennifer has decided to write her literary analysis paper on drug use in David Foster Wallace's novel, Infinite Jest (1996). She tries a few Google searches for Infinite Jest, drugs, and drug use, but she has trouble finding scholarly sources. She gives up on Google and moves on to EBSCO Academic Search Premier, one of the databases she heard about in a library instruction class. She runs a search for Infinite Jest and drug use, but she still can't find much.

  • Jennifer's Inference/Conclusion: I need to change my topic.
  • << Previous: Inferences
  • Next: Implications >>
  • Last Updated: Jul 10, 2023 11:50 AM
  • Librarian Login

logo

  • GMAT CLUB TESTS
  • FORUM QUIZ - NEW!
  • QUESTION BANKS
  • DECISION TRACKER
  • SCHOOL DISCUSSIONS
  • MARKETPLACE
  • T&C and Privacy Policy
  • GMAT Club Rules
  • Login Register Forgot password?
  • ${glob_var/L_LOGIN_LOGOUT}
  • Quick Search

How to Tackle Critical Reading Assumption Questions

Magoosh

Assumption questions ask you to find the unstated link between a question’s premise and its conclusion. Assumptions are crucial in understanding and refuting arguments, so they play a large role in two major Critical Reasoning question types . In this post, we’ll cover GMAT Critical Reasoning tips and practice questions to help you tackle assumption questions.

GMAT Critical Reasoning Tips: How to Tackle Assumption Questions

Luckily, arguments on GMAT Critical Reasoning questions are relatively formulaic, so let’s go over the basics first:

  • A premise is the starting point of the argument.
  • The conclusion is what the author wants you to believe by the end of the argument.
  • The assumption is the missing link between the premise and conclusion. Think of it like the linchpin holding the whole thing together. You can strengthen an argument by validating its assumption, or weaken the argument by denying the assumption.

Assumption questions will usually ask you, “Which would most strengthen the argument?” or “Which of the following would most weaken the argument?” (the latter is one of the most common on Critical Reasoning).

Make Your Assumption a General Statement

This is a crucial point to remember: assumptions are most often general statements , not specific statements. When you identify the assumption, you can omit any specific people, places, or items mentioned.

If my premise is “Fred has quality A,” and my conclusion is “Therefore, Fred has quality B,” Fred is a specific person that we can omit (sorry, Fred). The assumption would be something like “most/all folks who have quality A also have quality B.”

Identify the Assumption

Isolating an assumption is an important skill and one of our favorite GMAT Critical Reasoning tips. Let’s try it with this argument:

  • The premise is “Hawaii is a place with beautiful scenery.” (We can safely assume that at least 99 out of a hundred people would agree with that!) Hawaii is the specific, so you can omit that—the final premise has to do with a “place with beautiful scenery.”
  • The conclusion is “trouble concentrating.”
  • The assumption must provide a link. If we put those together with a strong logical connection, we get this assumption: “People in places with beautiful scenery generally have trouble concentrating.” Even though it’s a little absurd, that’s a possible way to state the assumption!

It would most strengthen this argument if one could somehow provide data or evidence supporting this assumption. This argument would be weakened if we could cite data or evidence that directly contradicts the assumption.

Now, consider an argument you’re more likely to see on the GMAT:

  • If we drop the specifics, the premise is about increasing spending on advertising, and the conclusion is: more new customers. An assumption would link these.
  • A very broad assumption: “Companies that increase what they spend on advertising generally see an increase in new customers.”
  • A slightly more specific assumption: “When companies in the steel industry increase advertising, this generally results in more new customers.”

This is a relatively poor argument, and if we were asked for a statement to weaken it, the best choice would be something that zeroed in on the assumption. For example, something like Studies of companies in the steel industry show little correlation between advertising dollars and new customers strikes right at the center of the argument.

Use the Negation Test to Verify the Assumption

If you want to verify that your assumption is really the correct one, you can use the Negation Test —put simply, try negating the statement and seeing if the conclusion is still true. If you haven’t tried the Negation Test yet (another of our key GMAT Critical Reasoning tips!), then I would definitely recommend checking out our post and studying this powerful technique for isolating assumptions of arguments.

Practice Questions and Explanations

1. Which of the following is an assumption that supports drawing the conclusion above from the reasons given for that conclusion?

Click here for the answer and video explanation!

2. Which of the following is an assumption that supports drawing the conclusion above from the reasons given for that conclusion?

3. Which one of the following is an assumption on which the conclusion depends?

If folks with Laestrygonian Disease cannot assimilate the Vitamin C in the rice, then it won’t help them, and eating the fortified rice will not provide them any particular benefit. If we negate this option, it shatters the argument. This is a true assumption.

(A) This may be true, although I am skeptical that any human-made improved food would be better than the fruits designed by Nature! Regardless, whether this is true or not does not have any bearing on how helpful the fortified rice will be for the folks with Laestrygonian Disease. This option is incorrect.

(B) This is intriguing. Let’s negate this. Suppose it were the exact same problem, say, the exact same missing enzyme, that made it impossible to digest both fruit and vitamin supplements. Then what? Would that mean they also couldn’t digest the fortified rice, or get the vitamin C they need from it? We cannot say. It’s conceivable that the argument could still work, so negating this does not destroy the argument. This is not an assumption.

(D) Let’s negate this. Suppose the fortified rice benefits everyone—even the no-carbs fanatic who hasn’t touched carbs in a decade: even when this person breaks his carb-fast and has the fortified rice, he has benefit from it. What then? Whether these other people benefit or not from the fortified rice has no bearing on whether it helps the folks with Laestrygonian Disease. This choice is incorrect.

(E) Let’s negate this. Suppose we can infused dozens of other vitamins and minerals into the rice, all with high nutritional yield. That would only be good for the folks with Laestrygonian Disease—the more vitamins, the better! It certainly would not impact whether these folks derived any benefit from the vitamin C in the rice. This choice is incorrect.

Final Thoughts

Assumption questions will require you to read closely, but with practice you can identify the missing link. For more GMAT Critical Reasoning tips, check out our introduction to the CR section .

The post How to Tackle Critical Reading Assumption Questions appeared first on Magoosh Blog — GMAT® Exam .

Magoosh

Published in GMAT , Magoosh , Blog and GMAT Prep

Want to create or adapt books like this? Learn more about how Pressbooks supports open publishing practices.

What is Critical Thinking?

What About Assumptions?

Assumptions are beliefs or ideas that are  believed to be true without proof or evidence and are used to support reasoning. This lack of verification can create bias when thinking critically. Like any human activity, the practice of critical thinking requires several basic assumptions to make sense. For people who don’t share these assumptions, the whole process can be experienced as confusing or nonsensical. Here is a partial list of assumptions that sometimes cause trouble for people new to critical thinking.

critical thinking assumption questions

  • In CT, reasoning implies evaluation, both individual (“You should recycle your aluminum!”) and collective (“We should abolish the death penalty!”). Each statement can be supported by reasons, and the reasons can be evaluated as better or worse. Although this should not be confused with opinions or facts .
  • In CT, “Truth is what is so  about something, the reality of the matter, as distinguished from what people wish  were so, believe  to be so, or assert to be so” (Ruggiero, 2015, p. 25)
  • When using critical thinking you should not contradict yourself. Contradictory statements , by definition, cannot all be true, and based on #3 above that means they can’t be partly true, or true to some people but not others.
  • Critical thinking requires judging other people’s opinions (along with our own!) – not in isolation, but in relation to each other.

Many people put the majority of their critical thinking energy into judging the thinking of those they disagree with (fast thinking). Our hope is that you will have come to understand that thinking carefully about your own beliefs is worth more of your time, and that you will have come to appreciate the vital importance of people who do not share your same ideas to your process of slow thinking.

Check Your Knowledge: Assumptions

Read the following statements and then determine the assumption.

The U.S. is overreacting to the growth of AI. Technology is meant to be utilized to its fullest.

“Eating healthy is important. Doctors and physical fitness advisors tell you about the advantages of health foods. Then why are these foods so expensive? Companies that sell these foods are raising prices for simple things such as fruits and vegetables….People want to be healthy but it seems that corporate America really doesn’t want to make that prospect cheap. You should avoid wasting money just to eat healthy; go buy cheap frozen vegetables at the grocery store.” (Browne & Keeley, 2018, p.56)

Taking an act or statement for granted (Merriam-Webster Online)

Judgements about good or bad, right or wrong

Evaluations for better or worse

A view or judgement

Something known or proven true

False statement

Critical Thinking in Academic Research - Second Edition Copyright © 2022 by Cindy Gruwell and Robin Ewing is licensed under a Creative Commons Attribution-ShareAlike 4.0 International License , except where otherwise noted.

Share This Book

Critical Thinking: Challenging Assumptions

  • 6 videos | 23m 6s
  • Includes Assessment
  • Earns a Badge
  • Certification CPE

WHAT YOU WILL LEARN

In this course.

  • Playable 1.  Critical Thinking: Challenging Assumptions 1m 16s Everyone makes assumptions. Some are useful, grounded in experience and evidence. But many others aren't and can lead to poor decision-making. Examining your assumptions is a crucial part of critical thinking. In this video, you'll learn the key concepts that will be covered in this course, including how to identify and investigate your own assumptions. FREE ACCESS
  • Playable 2.  Investigating Assumptions 5m 34s To move toward critical thinking, you must first acknowledge that everyone routinely makes assumptions. It's one of the ways that our brains process the huge amount of information we take in. But it’s not always the best way of making decisions or solving problems. In this video, you'll explore critical-thinking practices that create awareness in order to redirect automatic thinking into controlled thinking. FREE ACCESS
  • Locked 3.  Detecting Assumptions through Questioning 4m 33s Reaching successful conclusions requires developing your questioning skills. You can use different kinds of questions to uncover incorrect assumptions, open new lines of inquiry, and reveal the truth about a situation. In this video, you'll identify lines of questioning that help you detect assumptions that might impact decision-making. FREE ACCESS
  • Locked 4.  Drilling Down into an Assumption 5m 35s Investigating assumptions starts with exploring their origins to get insight into their validity. This involves asking a lot of questions, especially questions starting with “why.” In this video, you'll learn about techniques for helping you investigate assumptions thoroughly. FREE ACCESS
  • Locked 5.  Testing Assumptions Systematically 4m 49s As with any worthwhile investigation, looking into your assumptions takes time and effort. And when you’ve completed your inquiry, there’s still more work to do: In this video, you will learn the sequence of actions for testing your assumptions. FREE ACCESS
  • Locked 6.  Let's Review 1m 19s In this video, you will be able to review and reflect on what you've learned in the course Critical Thinking: Challenging Assumptions. FREE ACCESS

critical thinking assumption questions

EARN A DIGITAL BADGE WHEN YOU COMPLETE THIS COURSE

Skillsoft is providing you the opportunity to earn a digital badge upon successful completion on some of our courses, which can be shared on any social network or business platform.

YOU MIGHT ALSO LIKE

critical thinking assumption questions

PEOPLE WHO VIEWED THIS ALSO VIEWED THESE

critical thinking assumption questions

critical thinking assumption questions

  • Exam Prep >
  • Prepare for Business School >
  • Business School & Careers >
  • Explore Programs >
  • Connect with Schools >
  • How to Apply >
  • Help Center >
  • About the Exam
  • Register for the Exam
  • Plan for Exam Day
  • Prep for the Exam
  • About the Executive Assessment
  • Register for the Executive Assessment
  • Plan for Assessment Day
  • Prepare for the Assessment
  • NMAT by GMAC
  • Shop GMAT™ Official Prep
  • About GMAT™ Official Prep
  • Prep Strategies
  • Personalized Prep Plan
  • GMAT Mini Quiz
  • Executive Assessment Exam Prep
  • NMAT by GMAC Exam Prep

Prepare For Business School

  • Business Fundamentals
  • Skills Insight

Business School & Careers

  • Why Business School
  • Student Experience
  • Business Internships
  • B-School Go
  • Quiz: Are You Leadership Material?
  • MBA Return on Investment (ROI) Calculator
  • Estimate Your Salary
  • Success Stories
  • Diversity and Inclusion
  • Women in Business

Explore Programs

  • Top Business School Programs
  • Quiz: Which Post Graduate Program is Right for You?
  • Quiz: Find the Best Program for Your Personality
  • Business School Rankings
  • Business Master's Programs
  • MBA Programs
  • Study Destinations
  • Find Programs Near Me
  • Find MBA Programs
  • Find Master's Programs
  • Find Executive Programs
  • Find Online Programs

Connect with Schools

  • About GradSelect
  • Create a GradSelect Profile
  • Prep Yourself for B-School
  • Quiz: Can You Network Like An MBA?
  • Events Calendar
  • School Events
  • GMAC Tours Events
  • In-Person Events
  • Online Events

How to Apply

  • Apply to Programs
  • The Value of Assessments
  • Admissions Essays
  • Letters of Recommendation
  • Admissions Interviews
  • Scholarships and Financing
  • Quiz: What's Your Ideal Learning Style?

Help Center

  • Create Account
  • Exams & Exam Prep

GMAT Critical Reasoning – The Foundation of the Exam

Chris Kane

Chris Kane - Menlo Coaching

Chris Kane is a mba.com Featured Contributor.

Student Hallway Conference

No skill is assessed more on the GMAT than critical thinking. In one form or another, every question type – from Sentence Correction to Problem Solving – is cleverly evaluating your ability to remain critical and find flaws in your own approach to a question or within the information provided in that question. Why? Because the best managers and business executives are those who can quickly isolate problems and fix them efficiently and effectively.

The Critical Reasoning question type is, of course, the ultimate assessment of this essential business skill. GMAT Critical Reasoning questions are designed to evaluate a specific set of attributes relating to logic, and most students preparing for the test have never been exposed to this type of argument analysis. Broadly speaking, there are 3 types of Critical Reasoning questions you will encounter on the GMAT Verbal section:

  • Those in which you must attack a given argument or plan (called the stimulus) and then find the answer choice that exposes a flaw or improves a flaw within that stimulus.
  • Those in which you must analyze and describe the line of reasoning used within the stimulus.
  • Those in which you must supply a valid conclusion based on the information given in the stimulus.

For type 1 questions, there are several important subtypes that you will see described by test prep companies: strengthen, weaken, assumption, useful to evaluate, and explain the paradox. Type 2 questions are most commonly called method of reasoning questions (usually boldfaced questions), and type 3 are commonly referred to as inference or conclusion questions. Note: well over 75% of the Critical Reasoning GMAT questions you will see on your exam are type 1, so we will focus mainly on the strategies for that common type in this article.

Best Practices for GMAT Critical Reasoning

To succeed in Critical Reasoning, you must develop a structured set of best practices that help you quickly maneuver through the different tricks and traps you will encounter on these questions. Some of these tips, such as reading the question stem first, are universal across different test prep curricula, while others are more sophisticated.

1. Read the Question Stem First

Since the mindset and strategies required for these three types of Critical Reasoning questions (and even between the subtypes in category 1) are so different, it is essential that you read the question stem first to determine which type of question you are facing. As an example, consider the following two question stems, which can easily be confused:

  • Which of the following most supports the argument above?
  • Which of the following is most supported by the argument above?

In the first question stem, you are dealing with a strengthen question. You need to attack the given argument or plan and then find a new piece of information in the five answer choices that necessarily improves the given argument or plan.

In the second question stem, your approach is completely different! You take everything in the stimulus as fact, and then determine which of the answer choices is a valid conclusion based on that given information.

If you don’t read the question stem first, you are wasting valuable seconds in your initial reading of the given stimulus because you don’t know your goal, and you will thus face time pressure on the verbal section. Additionally, if you don’t categorize the question correctly, you are doomed to get it wrong, and GMAT question writers purposefully make this task difficult.

2. Master the Art of Deconstructing Arguments

Another core skill tested on the GMAT is your ability to contend with abstract presentation of information and concepts. Many Critical Reasoning questions contain extremely complicated arguments that are both abstract and difficult to comprehend.

One of the big mistakes students make in attacking Critical Reasoning questions is that they go to answer choices before they fully deconstruct and understand the given stimulus. Piling on additional information to something you don’t fully understand is a common mistake students make on the GMAT generally – don’t go to answer choices until you understand the given information and the goal of the question!

In our Menlo Coaching GMAT curriculum , we put a huge emphasis on teaching students how to properly deconstruct arguments. This is best done by reading the full argument (for type 1 and type 2 questions, in which you are presented with a full argument in the question stem) and then isolating the conclusion. To make sure you have actually found the proper conclusion, you “Ask Why?” to that conclusion and see if the question leads back to the premises within the argument. Once you have done all of this properly and fully understand the given stimulus, you should then move to the next step in your structured best practices.

3. Attack (and Anticipate)!

So at this point, you have categorized the question type and fully deconstructed the given stimulus; now the real game of GMAT Critical Reasoning begins!

For all type 1 questions, you will have a full argument or plan in the stimulus, and you need to attack that stimulus, finding any flaws or common fallacies within the line of reasoning. In 20 years of preparing students for this question type, I have noticed one universal attribute among the “masters” of GMAT Critical Reasoning: they find the flaws on their own first and then evaluate the answer choices. To some this may seem counterintuitive (shouldn’t you leverage the answers first?), but the reality for all type 1 questions is that the answer choices are more your enemy than your friend.

In general on the GMAT, an essential best practice is to leverage all answer choices and use them actively, but Critical Reasoning answer choices are notoriously manipulative. Incorrect answers tempt you with ideas and “flaws” that pollute your brain and keep you from isolating the true issues in the argument. By attacking the argument before digging into answers, you are in charge of finding the flaws (and you will hone this skill with lots of practice) and can avoid this “polluting of the mind” by clever incorrect answers.

When you do anticipate flaws in the stimulus (often called “pre-thinking” in test prep curricula), it is essential that you do so broadly: don’t anticipate exactly how the answer choices might relate to the flaw, but find the core issues in the argument that should be addressed within one of the answers, which often does so in an obtuse or confusing way. On the hardest type 1 questions, you will likely need to use some hints from the answers about what to consider in the stimulus, but you must do that carefully while you sort through a collection of tasty trap answers.

Importantly, this type of anticipation is really only used for type 1 questions. For method of reasoning questions, you will deconstruct the argument and perhaps find some flaws, but you need to use process of elimination and compare answers to see which one is accurately describing the argument or the boldfaced portions of that argument. For inference/conclusion questions, you cannot anticipate what valid conclusion the question writers might provide, so again you need to attack each answer choice using process of elimination until you find the one answer that is a valid conclusion.

In summary, if you want to improve accuracy and speed in GMAT Critical Reasoning, you must get comfortable attacking arguments on your own and recognizing the common logical fallacies found in these questions. In no particular order, here are some of the most common fallacies you will encounter in GMAT Critical Reasoning practice questions and on your exam:

  • Mistaking Correlation for Causation
  • Generalization
  • Numerical Data Flaws (using absolute number data when you should use percentage data, improper use of statistics, etc.)
  • Past Trends Don’t Guarantee Future Trends
  • Baseline Assumptions (forgetting about starting points and what might have been true in a previous time frame)

Make sure you understand these flaws deeply and get skilled at recognizing them in a variety of scenarios and presentations.

4. Analyze the Answers (and be ready for the more “sinister” tricks and traps in GMAT Critical Reasoning).

Now, you are fast at categorizing the question and deconstructing the stimulus. You have mastered the common logical fallacies used on GMAT Critical Reasoning questions and you are good at attacking arguments on your own. That means you will get most GMAT Critical Reasoning questions correct, right? Sadly, that is not enough!

The additional mechanisms used to make these questions hard are the most sinister and difficult to handle. These traps are mainly assessing who is paying careful attention to details and making sure their mind does not wander (again important skills in business!):

  • Wordplay. Wording scenarios and clever language construction are used to elicit misinterpretation and misreading by the test-taker. Often this involves negation or other forms of abstract wording.
  • Misdirection. Question writers insert captivating information (what I often call the “shiny penny”) to draw your focus away from what really matters, which may be quite boring..
  • Mental Inertia. This is a form of misdirection in which question writers use topics for which everyone has deeply preconceived notions, but the answer is contrary to what you expect.
  • Word Shifts. You will commonly see subtle word shifts that create gaps or flaws within an argument. As I joke in my classes, there is no such thing as “synonyms” in Critical Reasoning. If a word has been changed between premises or between premises and the conclusion, it is probably creating a gap within the argument.

Seeing through this type of difficulty and avoiding these common traps and tricks comes through doing lots of high quality official GMAT Critical Reasoning practice questions. After you get burned by these different cons enough times, you start to see the traps quickly and your percentile in GMAT Critical Reasoning will jump dramatically. Pattern recognition through completion of a high volume of official problems is essential to success in training for Critical Reasoning.

GMAT Critical Reasoning Practice Question

To conclude this article, let’s apply these best practices to one retired official Critical Reasoning question. Take around 2 minutes and try to solve the question on your own:

The program to control the entry of illegal drugs into the country was a failure in 1987. If the program had been successful, the wholesale price of most illegal drugs would not have dropped substantially in 1987.

The argument in the passage depends on which of the following assumptions?

A. The supply of illegal drugs dropped substantially in 1987.

B. The price paid for most illegal drugs by the average consumer did not drop substantially in 1987.

C. Domestic production of illegal drugs increased at a higher rate than did the entry of such drugs into the country.

D. The wholesale price of a few illegal drugs increased substantially in 1987.

E. A drop in demand for most illegal drugs in 1987 was not the sole cause of the drop in their wholesale price.

Explanation

This problem is a great example of the wordplay and misdirection that I highlighted previously. The argument itself isn’t that difficult to understand but the answers are manipulative and the wording tricky. Let’s apply all the previously discussed GMAT Critical Reasoning tips in a methodical way to find the correct answer:

  • Categorize. The question is clearly an assumption question, a type of strengthen question in which you must isolate a necessary premise in the 5 answer choices. The correct answer will necessarily improve the quality of the argument.
  • Deconstruct. Since this is a relatively short and simple argument, it is easy to deconstruct. You should first isolate the conclusion – ”The program to control entry of illegal drugs was a failure in 1987” – and then “ask why?” to build back any given premises. Why was it a failure? Because if it had been a success, the wholesale price would not have dropped substantially in 1987. As is often true in assumption questions, you must deal with negation both in the given argument and in some of the answer choices. To help better understand the argument, simplify the given premise using affirmative language (but be careful not to change the meaning!). “Because if the program had been a success, the price would have remained the same or gone up (or perhaps just dropped a little)”
  • Attack and Anticipate. This argument presents a classic flaw in which alternative explanations are ignored. The argument relies on the idea that if the price went down substantially, it must have been because supply increased from the entry of illegal drugs and thus the program failed. But wait a second what else could have caused the price to drop substantially? If demand for illegal drugs dropped in 1987, then the price could have gone down even if the program was successfully preventing the entry of drugs. Alternatively, domestic production could have increased the supply of available drugs and the price could have gone down even with successful control of illegal entry from other countries. With those two flaws in mind, carefully assess the answers.
  • Analyze the Answers (while looking out for wording tricks and more sinister traps).

(A) The correct answer should support the idea that the program was a failure. If anything, this statement does the opposite! If the supply dropped, then likely the program was a success, not a failure (but we don’t even know because the decrease in supply could have been related to domestic sources). On most strengthen questions, at least one trap answer will weaken the argument, particularly when negation is present, because test-takers get confused about their goal with the argument.

(B) This tricky incorrect answer contains the negation you often see in GMAT Critical Reasoning assumption questions. Another common Critical Reasoning tip you will see across most test prep curricula is the assumption-negation technique. In short, this technique can be used to deal with difficult negation in answer choices for assumption questions. If you negate an essential premise (which is the correct answer in an assumption question) then it should destroy the argument. This allows you to take any answer choice presented negatively and read it affirmatively to see if it destroys the argument. Let’s do that here:

The price paid for most illegal drugs by the average consumer did not drop substantially in 1987.

So if the price paid did go down, this supports the idea that the program was a failure! If it was correct, this negated version should destroy the argument. The human brain does not deal with negation well, so it is much easier to read answers like this affirmatively to see if they destroy the argument. Both of the first two answers are wrong essentially because they do the opposite of what they should. Also, you should note that this answer is about the price paid by consumers while the argument is using wholesale prices for its evidence a classic word shift!

(C) To make the argument that the program was a failure, you do not need to know that the domestic rate increased more than the foreign entry rate. The program would be a failure even if the domestic rate increase was less than the foreign entry rate increase. This comparison is irrelevant to the given conclusion but since it broadly addresses the domestic vs. foreign issue that you should have considered after your initial reading, some people will still pick it.

(D) The given argument involves “the wholesale price of most illegal drugs” while this answer just discusses a few illegal drugs. A few illegal drugs could increase while most drop with no effect on this argument. Note: if anything, this statement would again weaken the conclusion when we are trying to improve it. An increase in prices suggests the program might have been a success, not a failure.

Correct Answer

(E) Jackpot! Since this answer is presented negatively, let’s apply the assumption-negation technique and see if this destroys the argument.

A drop in demand for most illegal drugs in 1987 was not the sole cause of the drop in their wholesale price.

So, if a drop in demand WAS the sole cause of the drop in their wholesale price, this argument is completely destroyed because the price drop was not due to a failure of the drug entry program. This correct answer essentially removes a flaw, that if true, would be very problematic for the given conclusion.

Final Takeaways

To succeed in GMAT Critical Reasoning you need to follow a structured set of best practices. As you can see from this one example, the difficulty can be more from the negation and wording than from a difficult-to-find flaw or confusing argument. The most important skills to develop while training with official GMAT Critical Reasoning practice problems are the following:

  • Quickly be able to deconstruct and understand given arguments.
  • Attack arguments efficiently and be able to find all broad flaws within a line of reasoning on your own.
  • Learn how to contend with the difficult tricks and traps relating to wordplay and abstract presentation.

When those skills are mastered, you will see your CR percentile jump considerably AND you will be able to complete CR questions faster, allowing for less time pressure on the GMAT verbal section.

Chris Kane is an mba.com Featured Contributor.  

The Magoosh logo.

Introduction to GMAT Critical Reasoning (with Practice Questions)

A female student thinking about her test questions

This post was updated in 2024 for the new GMAT.

What is GMAT critical reasoning?

GMAT Critical Reasoning is one of the two question types found in the GMAT Verbal section . All CR questions contain a prompt (usually 100 words or fewer) that presents some sort of argument, which is followed by a question stem and 5 answer choices. The questions revolve around a logical analysis of the core argument and might involve strengthening it, weakening it, finding its underlying assumption, etc. GMAT Critical Reasoning tests your critical thinking and logic skills more than your reading skills.

Logical reasoning is also one of the most important skills in the business world. Whether to decide upon the best sales strategy, to understand what motivates your customers, or to navigate investor concerns, well presented, logical arguments are the bedrock of successful business management. An effective manager demonstrates skills in addressing the following: How would I strengthen or weaken this argument? What is the assumption of this argument? What further evidence would I need to evaluate this argument? In other words, a real-life manager needs to apply all the skills required for success on GMAT Critical Reasoning questions.

The 8 Critical Reasoning Question Types

The eight broad categories of GMAT Critical Reasoning questions are:

1) weaken the argument /find the flaw in the argument 2) strengthen the argument 3) find the assumption 4) draw inference/conclusion 5) structure of the argument, including boldface structure questions and dialogue structure questions 6) paradox 7) evaluate the conclusion 8 ) complete the argument

Types #1-4 account for most GMAT CR questions. You can find out more about each one of these types in the linked blog articles.

Top Tips for Improving in GMAT Critical Reasoning

Tip #1 – read the question first.

Read the question stem before reading the argument . This will help you know which type of question you are going to have to answer. Then, you can read the argument with that question in mind.

Tip # 2 – Know the Different Elements of an Argument

In reality, every argument has three key components:

  • An assumption, which is usually not stated in the argument but must be true for the conclusion to be valid. In other words, if the assumption is found to be false, the entire argument will fall apart.

If you have a mathematical bent of mind, think of these components through the following equation:

Premise + (Unstated) Assumption + (extra context/background info) = Conclusion

The two that are usually easier to identify are the conclusion and the premise. But! It’s important to note that more complex CR question types might not even have a stated conclusion or easily identifiable premise. However, when those elements are present, it is important to think about the underlying assumption that is holding the argument together. Your job is never to question the facts of the argument. You have to take anything stated as truth. Your job is to find the unstated assumption and question the gap between the facts and the conclusion. Ask yourself: What did the author need to be true in order to reach the conclusion?

To become good at solving CR questions, it is important to become good at analyzing the argument and identifying all these components of the argument. You can find more details regarding assumption questions here .

Tip #3 – Know What You’re Looking For Beforehand

In all Critical Reasoning questions, the GMAT gives one correct answer and four tempting and potentially confusing statements for the other choices. Folks who read the argument & question and then wander aimlessly into the answer choices without any further thought are asking to be perplexed, and, chances are, they spend much longer than necessary on the questions.

Go into the question with an idea of what you seek. For types #1-3, the best thing to do is to find the assumption of the argument — reaffirming or undercutting the assumption of an argument is the most powerful way to strengthen or weaken it. Finding the assumption may also be helpful in finding the flaw of the argument (if the flaw is a faulty assumption).

For the other question types, you will be less able to predict what the answer will be; still, formulating the task in your own words will help you. In your own words, what is the structure of the argument? What is the paradox that needs to be resolved? What kind of information would be required to evaluate the conclusion? etc. The more clearly you understand what type of information or argument will satisfy the question, the more quickly you will find it.

Tip #4: Read EXACTLY what is written

The majority of mistakes in the critical reasoning section often boil down to the same fundamental error that students make: misreading the argument, question stem, or one of the choices. Furthermore, the argument is limited to the topic presented. Any answer choice that alters or moves away from the topic of the argument is a trap.

It is extremely important to take your time, stay engaged with the argument and read EXACTLY what is written without trying to paraphrase it. A single word can change the meaning of the conclusion or what piece of information can make that conclusion invalid. It might seem beneficial to simplify certain details or ignore certain modifiers that seem like ‘extra’ information for the argument, but that is what can get you into trouble. Reading carefully and noticing every modifier or any extra information in the passage is the best way to avoid mistakes.

Tip #5: Look for Four Wrong Answers, Not the Correct One.

Almost all Critical Reasoning questions will have at least a couple of choices that are clearly incorrect. When struggling through the confusing options, your focus should be to first find those low fruits and eliminate them. In the first pass, only eliminate the option choices that you are completely sure are incorrect. You can always go through the options again and iteratively make eliminations to find four incorrect choices. This will also allow you to “narrow the field” and focus on the more difficult or confusing choices that require slightly more focus and attention. Once you have done that, the last remaining choice, no matter how confusing or how strange, has to be the correct answer.

GMAT Critical Reading Practice Questions (with Explanations)

  • More GMAT Critical Reasoning Questions
  • Megalimpet is a nationwide owner of office space. They have major office buildings in the downtowns of several cities in the 48 lower states, and rent this space to individual companies. Megalimpet office spaces vary from small office to large suites, and every space has custom-designed wall-to-wall carpeting. The carpet in several Megalimpet facilities needed replacing. The winning bid for the nationwide carpet replacement was submitted by Bathyderm Carpet Company (BCC). The bid contract involves all delivery costs, all installation, and any ongoing maintenance and upkeep while the carpet is under the three-year warranty. Both BCC executives and independent consultants they hired felt BCC would be able to perform all these services for far less than their bid price; these circumstances would allow BCC to reap a considerable profit.   Which of the following, if true, most calls in question the argument that BCC will make a large profit from this contract with Megalimpet?
  • Click here for a video answer and explanation to the first of our GMAT Critical Reasoning Questions!

From all the evidence given, it seems that BCC (and the independent consultants) have taken all costs into account, and the analysis reveals that they will reap a considerable profit. In order to call this into question, we have to come up with some major unanticipated cost that would not be something already considered in this analysis.

(D) is the credited answer. First of all, laser printers and photocopiers are very common devices in office spaces, so we good reason to think that many of Megalimpet’s tenants will use these. If the toner degrades the carpet, that’s a huge additional expense for BCC, because their contract includes “ongoing maintenance” — i.e. replacing any carpet that needs replacing. Finally, nothing in the argument stem gives us any indication that this problem was on anybody’s radar, so this well could be an unexpected or unanticipated expense for BCC. Therefore, it most calls into question the idea that BCC will make a huge profit.

(A) & (C) are all expenses that would have been very clear to BCC and to its independent consultants, and therefore all of these would have had to have been taken into account when the financial analysis of the bid was made. There is no reason any of these expenses would be unanticipated.

(B) speaks to BCC previous experience, which, if anything, would tend to suggest they know what they are talking about. If anything, this would tend to strengthen the argument, not weaken it.

(E) only compares BCC to the second lowest bid, but we have no idea about that company, what it did or did not take into account in their bid, and what their overall costs might be. There are too many unknowns for this piece of information, by itself, to have any substantial impact on the argument.

  • A minor league baseball franchise experienced a drop in attendance this week after they suffered three losses by margins of ten runs or more last week. Many spectators of those games wrote letters to the editors of the local sporting news, complaining of the poor play of the team in those three losses. Nevertheless, the front office of this baseball franchise maintains that the team’s poor play in those three losses has nothing to do with this week’s decline in attendance.   Which of the following, if true, most strongly supports the position held by the front office of the baseball franchise?
  • Click here for a video answer and explanation to GMAT Critical Reasoning Question 2!

The statement by the front office of the baseball franchise seems, on the surface, not to take the basic facts into account. If we want to strengthen this position, there must be some alternate explanation for the drop in attendance.

Part of strengthening the franchise’s position would be weakening the original position: namely, that the team’s poor play explains the drop in attendance.

(C) is the credited answer. If other minor league teams also experience a drop this week, there must be something global in this market affecting all teams. We don’t know what this factor is, but it’s something that touches all teams, not just those that played bad last week. This provides a cogent alternative explanation, even though we don’t know the specific nature of the factor causing the drop in attendance.

Both (A) & (D) strengthen the original position, namely, that the team’s poor play explains the drop in attendance. In order to strengthen the baseball franchise’s position, we have to weaken this original position.

Choice (B) essentially accuses the baseball franchise of lying, or at least bluffing, which hardly strengthens their position.

Choice (E) simply adds to the paradox: if the closest MLB team is far away and folks typically don’t go there, then there would be more demand for the local minor league baseball. Given that demands, a drop in attendance doesn’t make as much sense. This choice adds to the confusing without explaining anything.

  • In a few recent cases, some teenagers with advanced programming abilities used a new programming language, FANTOD, to hack into ETS and change their own SAT scores. All of the teenagers convicted of this crime were highly skilled in programming FANTOD. In light of these cases, some colleges have discounted the official SAT scores of applicants with a knowledge of FANTOD, and have required them to take special admission tests in supervised conditions on their own campuses.   Which of following conclusions can most properly be drawn from the information above?

Click here for a video answer and explanation to GMAT Critical Reasoning Question 3!

The evidence says: all the ETS hackers were FANTOD programmers. What the colleges seem to be assuming is the converse: all FANTOD programmers are hackers. Of course, there is no direct evidence for this converse. Presumably there are some students who learn FANTOD in good faith and who are not hackers, but because of the assumption the colleges are making, these students are faced with extra challenges, such as having their justly achieved SAT scores disregarded and being forced to take additional admission tests.

(C) is the credited answer. Since there is no evidence for the converse statement, we have reason to believe there are FANTOD programmers who are entirely innocent of any hacking, yet those very students will have their perfectly valid SAT scores dismissed and will have to take a new test to achieve admission: this certainly would not be fun, would not be fair, and could place them at a disadvantage with respect to all the non-programming students who could just take the ordinary SATs and be done with all testing.

(A) assumes too much based on the information provided in the prompt. Specifically, we only know about a specific group of those with FANTOD knowledge: those who used it to hack into ETS. Therefore, we cannot make any airtight conclusions about “most people”. It is very possible that most people who know FANTOD use it for purposes other than hacking.

(B) might be true, but it’s much too broad. This is about the much larger issue of what is the best way for colleges to determine who should be admitted. This entire argument is focused quite specifically on the FANTOD programmers and the issues associated with them.

We have absolutely no evidence for (D). All we know is that, whatever scores those hackers achieved on the real SAT, they falsified the records to make them higher. We don’t know if those scores were already high, and we certainly can draw no conclusion about all the students who know how to program in FANTOD who are not hackers. In fact, one might suspect the opposite, that folks bright enough to figure out this sophisticated programming language might be more intelligent and more successful on average, but even that we strictly can’t assume. Therefore, we can’t draw a clear conclusion about this.

(E) is a tricky one. We are told that some colleges took a certain set of special measures. We are given no information on what the other colleges did. Did they take another set of special measures? Did they not address the issue at all? We don’t know. Therefore, we can’t draw a clear conclusion along these lines.

  • In the twentieth century, the visual arts have embarked on major experimentation, from cubism to expressionism. While tastes always vary, there are certainly some people who find beautiful objects of each of the art movements of the first half of the twentieth century. In the latter half of the twentieth century, though, most works are so abstract or shocking that neither the critic nor the general public uses the word “beautiful” to describe them: indeed, sometimes late twentieth-century artists have, as one of their expressed goals, the creation of a work that no one could find beautiful. Whatever these artists are creating may be intellectually engaging at some level, but it is no longer art.   Which of the following is an assumption that supports drawing the conclusion above from the reasons given for that conclusion?

Click here for a video answer and explanation to GMAT Critical Reasoning Question 4!

The argument makes a number of factual statements. Art in the first half of the 20th century are, or could be considered, beautiful. Works by artists in the latter half of the 20th century are not supposed to be beautiful, and even, are supposed to be devoid of beauty. Then the argument draws a bold powerful conclusion: therefore, they are not art! The assumption seems to be something that links beauty to whether something qualifies as art. We definitely need an answer to speak to the question: what does, or doesn’t, qualify as art?

(C) is credited answer. If something needs to be beautiful, or potentially beautiful, to qualify as art, then this would explain that works that “no one could find beautiful” would fall outside the author’s definition of art.

The other answers are all quite tempting, because we could imagine an art professor or someone in an art class arguing for any one of them.

(A) is irrelevant. Critics & the general public might have different appraisals, but what one or the other thinks does not, in and of itself, seem to determine whether something is art.

(B) is also irrelevant: who determines the meaning is a separate question from whether the work qualifies at art in the first place. (BTW, exceedingly few modern critics would accept the interpretive idea contained in choice .)

(D) is undeniably true, but not relevant: again: it provides no standard by which we could say the former objects are art and the latter objects aren’t.

(E) is a far-flung idea, unrelated to the discussion. The passage doesn’t address the issue of whether any works of art are intellectually engaging.

  • The National Farm Administration (NFA) has been concerned over the last decade with the struggles of barley growers.   Plan : In an effort to support these barley growers, two years ago, the NFA began a program of sending them, each autumn, a free special mix of fertilizer and enzymes designed to multiply barley yield, to be applied the following spring during first growth. This mix had been stunningly successful in multiplying the yield of barley in laboratory conditions.   Results : Most barley growers reported little change in their economic status over this two year period.   Further information : All barley growers received the shipments, and all used them. Weather conditions have been fair to optimal for barley growth over the past two years.   In light of the further information, which of the following, if true, does most to explain the result that followed the implementation of the plan?

Click here for a video answer and explanation to GMAT Critical Reasoning Question 5!

  • When, on a particular shopping trip, a consumer purchases an item which he previously had no intention of purchasing, this sale is called an “impulse purchase.” The objects of impulse purchases are occasionally essential items (i.e. items that satisfy basic subsistence needs), but much more frequently are luxury or non-essential items. Researchers have determined that, at the end of a shopping trip, a consumer is much more excited if she has bought a luxury item on an impulse purchase, than if she had made no impulse purchases.   If the information above is true, and if the researchers’ investigation was properly conducted, then which of the following must also be true?

The credited answer is choice (D) . If the researcher was able to conclude anything about how an impulse purchase made someone feel, then the researcher first had to know that it was an impulse purchase, that is, that the purchase was not planned. If the researcher had no way to determine whether a purchase was planned or unplanned, then the researcher would have no way of determining which purchases were impulse purchases.

We know the consumer find the impulse purchase of a luxury item more exciting than the planned purchases. We don’t necessarily know how exciting the impulse purchase of an essential need is—maybe it’s less exciting than the impulse purchase of a luxury item, or maybe it’s just as exciting. We suspect from real life that this may be true, but we cannot determine this from information in the prompt, so it can’t be the answer to a “must be true question.” Thus, choice (A) is incorrect.

We only know about the excitement brought about by an impulse purchase of a luxury item, but we have no information about what happens if a purchase is planned but not made. Choice (B) inappropriate extends the pattern into situations the prompt doesn’t cover at all. Choice (B) is incorrect.

We know that the impulse purchase of a luxury item is exciting, but we don’t know whether this is sufficient inducement for a person seeking excitement to make this kind of purchase frequently. The expense, for example, might be a mitigating factor. We can conclude nothing for certain about this, so choice (C) is incorrect.

This is a tempting one—we certainly might suspect that the luxury items of higher price would be bought as impulse purchases less frequently. We might suspect this, but notice that the prompt says nothing about high price vs. low price items. This answer choice invites us to bring in irrelevant outside knowledge, so, like (A), it can’t be the answer to a “must be true question.” Choice (E) is incorrect.

  • Over the past ten years, the population of Dismaston has grown five times as large as it was. During this time, the average income in the city has risen substantially, and a tremendous amount of capital has flowed into city. An independent audit found that, somewhat surprisingly, the number of violent felonies reported per year is now lower than it was ten years ago.   Each of the following statements below, if true, would explain the somewhat surprising finding EXCEPT:

This is an EXCEPT question. Four of the answer will be perfectly valid explanations, and these will be incorrect. One of the answers will not be good explanation—either it will be irrelevant, or it may even suggest a rise instead of a decline; this oddball choice will be the correct answer.

The credited answer is (B) . The new filing system, in essence, never misses the report of a violent crime. This at least implies that perhaps the previous filing system missed some violent crimes on occasion—for whatever reason, some violent crimes that took place slipped through the cracks and failed to be reported. Well, if we were not reporting everything before, and are reporting everything now, if anything this might suggest an increase in the number of reported violent crimes. It most certainly would not, by itself, explain a decrease. This is not in any way a good explanation, so this the correct answer.

We know, over the past decade, “ the average income in the city has risen substantially ” and “ a tremendous amount of capital has flowed into city ,” both of which indicate conditions of prosperity. Therefore, according to choice (A), white-collar crimes would increase, and street-crimes would decrease, with a concomitant drop in violent crimes. Choice (A) is a valid explanation, so it’s an incorrect answer.

If the state kept convicts in jail longer, that would mean fewer of them would be back out on the streets committing felonies, most of which are violent. Therefore, it would lead to a drop in the number of violent crimes. Choice (C) is a valid explanation, so it’s an incorrect answer.

Better police and better crime detection means more arrests and fewer violent crimes. Therefore, it would lead to a drop in the number of violent crimes. Choice (D) is a valid explanation, so it’s an incorrect answer.

Better lighting at night and security cameras have some effect in reducing crime. Choice (E) is a valid explanation, so it’s an incorrect answer.

  • Archeologists have discovered three sites showing conclusive evidence for the mastery of fire in Tanzania, from a period slightly after the time that Homo habilis was present in Africa. These sites clearly were founded by Homo erectus , the descendent species of Homo habilis that migrated north, out of Africa and into Asia. Homo erectus was known to have mastered fire, from ample evidence at sites in Asia. There is no reason to attribute mastery of fire to Homo ergaster , the descendent species of Homo habilis that remained in Africa.   Which of the following is an assumption on which the argument depends?

Remains of prehistoric fire were found in Tanzania . The author says that Homo erectus made these fires, and that there’s no reason to assume Homo ergaster did. What is a necessary assumption?

The credited answer is choice (A) . Homo erectus had to be as far south as Tanzania—if they were not, there would be no way they could have made those fires there, which would seem to indicate that Homo ergaster made them after all. Negating this statement devastates the argument, which is a confirmation that we have an assumption.

Whatever might have caused Homo erectus to master fire doesn’t clarify who made those fires in Tanzania: Homo erectus or Homo ergaster ? Choice (B) is not correct.

Suppose Homo ergaster would have derived as much benefit from the master of fire as did the Homo erectus , or even more benefit. That fact, by itself, would imply nothing about which one of these species created those fires in Tanzania. Denying this doesn’t change the validity of the argument. Choice (C) is not correct.

Choice (D) is intriguing, because it may be true. Both Homo erectus and Homo ergaster evolved from Homo habilis , so it’s quite likely that the Homo habilis was the sole source of cultural knowledge for either of these species. BUT, we know that Homo erectus , presumably without the benefit of cultural knowledge about fire, was able to master fire. If Homo erectus did that, why couldn’t Homo ergaster ? In other words, the limits of the cultural knowledge inherited does not necessarily set limits on what these human species could achieve. Therefore, we can draw no conclusion with respect to this argument. Choice (D) is not correct.

If Choice (E) were true, it would support the argument, but a supporting statement is not necessarily an assumption. We have to use the Negation Test . Suppose Homo ergaster was all over in Tanzania, before & during & after the time that those fires were created. Would that prove Homo ergaster started those fires? Not necessarily. It could still be true that both Homo ergaster and Homo erectus occupied that region, that only the latter had mastered fire, and therefore, that the later had to start those fires in Tanzania. Thus, we can deny choice and it doesn’t necessarily contradict the argument. Therefore, it is not an assumption. Choice (E) is not correct.

  • Five years ago, the town of Bayside, in the Katonic River Valley, had catastrophic flooding one spring, and consequently, most insurers now refuse to write flood insurance for houses in Bayside. The town of Dryadia, in the Phemptic River Valley, is much like Bayside in its proximity to a similar river at an almost identical point in the river valley. We can conclude that the only reason the same insurers do not write flood insurance for houses in Dryadia either is its similarity to Bayside in terms of where it is situated in the river valley.   Which of the following, if true, would most seriously undermine the argument?

The credited answer is choice (D) . If the town of Dryadia really does flood, then that’s the reason insurers won’t write flood insurance for it! Therefore, the “ only reason ” cannot be “ its similarity to Bayside in terms of where it is situated in the river valley .” Choice (D) , if true, obliterates the argument, so this is the best answer.

The argument say that “ most insurers ” don’t write flood insurance in either town, but if most don’t , this implies that some do . Therefore, choice (A) is actually expected from the argument and does not challenge it at all. Choice (A) is incorrect.

Choice (B) would not be surprising and could be perfectly consistent with the argument. We know Bayside had “ catastrophic flooding “, but we don’t know for a fact that every single house was flooded—maybe or maybe not. If some houses were not flooded, it sounds as if the insurers don’t write flood insurance for any house in Bayside, so even those houses that never flooded could not buy flood insurance. Therefore, this would validate (B) without threatening the argument in any way. Choice (B) is incorrect.

Choice (C) is irrelevant. Even if no resident in absolutely any other town up and down the Katonic River Valley can buy flood insurance, that doesn’t necessarily shed light on why folks in a town in a completely different river valley can’t buy insurance. Choice (C) is incorrect.

Choice (E) is too general and vague. Yes, perhaps there are many ways a house can be flooded, and correspondingly, perhaps there are many reasons why an insurer would deny any particular house flood insurance. Even if this is true, it doesn’t shed any light on exactly why the folks in Dryadia have trouble getting flood insurance. Choice (E) is suggestive, but it doesn’t actually tell us anything. Choice (E) is incorrect.

Where to Find More GMAT Critical Reasoning Questions

While 10 Critical Reasoning questions are a great start to your practice, they’re definitely not all you should do before test day! Here are a few resources you can go through to get even more practice with different CR question types before test day.

  • Critical Reasoning Questions: Boldface Structure
  • Critical Reasoning Questions: Paradox
  • Critical Reasoning Questions: Conclusion/Inference

Critical Reasoning questions challenge your thinking skills and ability to critically analyze arguments. They need great attention to detail and are immune to most shortcuts or gimmicks. Follow the above discussed tips to understand what is tested in these questions and master the GMAT CR.

Looking to put your Verbal skills to the test? Check out the FREE Magoosh GMAT practice test with with accurate score prediction and subject-by-subject performance breakdown! Take just the Verbal section for 45 minutes or the whole assessment test for 2 hours and 15 minutes.

Happy studying!”

Ayush Verma

Ayush is a Test Prep Expert and Application Coach actively involved in the Test Prep and Application Consulting space for several years now. He is a GMAT 99 Percentiler and has extensive experience in delivering private tutoring sessions for GMAT, GRE, and SAT exams. Ayush has a Bachelor’s in Computer Science and an MBA in Strategy from the Indian School of Business. He is an ardent ManUnited fan and when he is not helping students understand that tricky GMAT question or write that dreaded Why MBA answer, he would be likely cheering for his team at Old Trafford (GGMU). To connect with him, feel free to reach out to him via LinkedIn or his website Test Prep Buddy .

View all posts

More from Magoosh

critical thinking assumption questions

Leave a Comment

Please leave any questions or suggestions in the comments, we try our best to respond within a few days! Your email address will not be published.

Leave a Reply Cancel reply

Your email address will not be published. Required fields are marked *

11 responses to “Introduction to GMAT Critical Reasoning (with Practice Questions)”

Jack Avatar

For question 2 above regarding BCC’s bid for carpeting Megalimpet offices, E strikes me as a much better answer than D.

Yes, D describes what sounds like an expensive maintenance cost, but it also asserts that this is would be standard/common for any other offices that BCC would have carpeted. We have little reason to believe that BCC would not have taken this cost into account. We’d have to add the additional assumption that BCC has either never provided this service for an office location, or that this is a new, untested carpeting material — quite a stretch.

However, for E, the evidence that, in a competitive bidding situation, the closest bid was 50% higher, does indicate that BCC must have very different assumptions than any other competing company, which indicates more clearly that something is awry in their assumptions.

Magoosh Expert

I like the deep critical thought you’ve given to this. You’re right that this could indicate something is awry if BCC is bidding very differently than the other companies. However, there’s a wrinkle there that makes the implications of answer choice (E) a bit unclear: Is BCC the one with incorrect assumptions, or are the other bidders the ones whose assumptions are considerably “off”? (E) provides no clear answer there, so it’s simply too weak to be a viable correct choice.

Martin Avatar

Hi Mike! Thank you very much for this post, it is really helpful.

I have a question regarding question 6. The way I thought about it is that the explanation for the paradox should find an explanation about why the plan had no effect on barley growers result.

In that sense, I chose answer B. Reason: While transporting the mix, because of some effect with heat or not having the proper storage conditions, the mix could have been altered and thus loose its properties. Barley growers applied mix, getting no results at all vs. previous condition. While considering answer C my thought was that if the mix helped a harmful bacteria to thrive, then the observed results after applying the plan should have been much worse than the present situation of barley growers, not just remain unchanged. Maybe I went a little to far with the storage reasoning, but I didn´t feel comfortable with the bacteria explanation because I would have expected much worse results than the observed.

Could you please explain find the flaw in this approach that is obviously wrong?

Question 6 is an interesting and very tricky one, Martin. Your reasoning on B could work in theory. The problem is that for GMAT CR, you can’t make that much of a stretch. If the answer choice doesn’t strongly imply somethign could be the case, then you can’t assume it’s a possibility.

Howe does this practice apply to (B)? While B talks about how transport of the barley growth mix is shipped form Wisconsin, and “sometimes” transit takes “as much as” 4-5 days. But that’s very tentative language. It doesn’t say how often transport takes 4-5 days, versus how often transit times are much shorter. The answer choice also doesn’t mention anything about transportation costs, in terms of money or in terms of the mix degrading during the transit period. Without these factors mentioned in the answer or the original paragraph, you can’t know if most transport times are even all that long, much less know whether transport affects the quality of the mix.

On the other hand, answer (C) directly references a specific problem that could affect barley yields. If you just look at answer C’s statement that the growth mix promotes a bacteria that harms younger barley plants, it would seem that barley growers’ economic problems would get worse, rather than remain the same. However, you should always combine info in the answer with info in the paragraph. The paragraph says that the mix also helps barley to grow more prolifically. So it’s distinctly possible that the mix’s promotion of barley growth and bacteria growth cancel each other out. The farmers might make gains in the growth of some plants, while also losing other plants to the bacteria that the mix also promotes. The net result could easily be no change in the amount that barley farmers struggle. So (C) is the best possible explanation for the paradox.

Abhishek Kumar Avatar

Hi, Mike I had purchased Manhattan Test series of 6 CATs and am some what getting average scores of 630-650 however while independetly trying the gmat questions i.e in your this test i got all 10 correct or from the books/ study material I’m getting most of them correct. Can you let me know how accurate the scores of these adaptive tests are? I have official Gmac Free test also pending and I can use that but unfortunately I had used one early in my preparation and secon I wish to use few days prior to my actual GMAT to build more on it. Can you help? Also do you have some very short program to boost verbal score I had booked My GMAT for 19th of this month however not gaining confidence in verbal section were I’m gettin score in the range of 25- 33.

Hi Abhishek,

Although I cannot speak on behalf of Manhattan, I will say that Magoosh mock exams are comparable to the actual exam. Still, the best gauge of your progress will be the GMATPrep exams from the makers of the actual exam. In addition, our Magoosh content will be helpful for your verbal score; however, we don’t have a specific short boost program for verbal. Honestly, your improvement will be based upon your effort.

Junjun Avatar

Great Post, I find one of the most difficult part for CR questions on the real test is the question stem is so hard to understand compared those on OG.

paresh Avatar

i have read both Powerscore cr bible and Manhattan cr. I found Powerscore cr too lengthy and boring. Manhattan cr was very comprehensive and organised.

Please let me know if Magoosh CR would be any different from Manhattan CR.

After reading both books I attempted the untimed OG test and scored badly in it.

Please let me know how to improve my CR skills.

Thanks, Paresh

Mike MᶜGarry

Dear Paresh, Magoosh has a video lesson series on CR that has somewhat different emphases from MGMAT’s approach, but of course, we cover much of the same material. I think Magoosh’s really strength lies in the video explanations for each question. In Magoosh, each GMAT CR question has its own explanation — of course, this video explains the individual question, but in doing so, it clarifies important points of content and general strategy. Taken as a whole, I believe what Magoosh offers is just as good, if not better, than what MGMAT has to offer. Does all this help? Mike 🙂

Thanks for the valuable reply.

Regards, Paresh

Dear Paresh, You are quite welcome. Best of luck to you, my friend. Mike 🙂

  • Debate Activities and Role-Play Scenarios
  • Reducing Air Pollution Levels
  • Exploring Brainwalking: A Creative Problem-Solving Technique
  • Critical Path Analysis: A Comprehensive Guide
  • Analytical problem solving
  • Identifying root causes
  • Analyzing consequences
  • Brainstorming solutions
  • Heuristic problem solving
  • Using analogies
  • Applying existing solutions
  • Trial and error
  • Creative problem solving
  • Mind mapping
  • Brainstorming
  • Lateral thinking
  • Research skills
  • Interpreting information
  • Data collection and analysis
  • Identifying patterns
  • Critical thinking skills
  • Recognizing bias
  • Analyzing arguments logically
  • Questioning assumptions
  • Communication skills
  • Negotiation and compromise
  • Listening skills
  • Explaining ideas clearly
  • Planning techniques
  • SWOT analysis
  • Gantt charting
  • Critical path analysis
  • Decision making techniques
  • Force field analysis
  • Paired comparison analysis
  • Cost-benefit analysis
  • Root cause analysis
  • Five whys technique
  • Fault tree analysis
  • Cause and effect diagrams
  • Brainstorming techniques
  • Brainwriting
  • Brainwalking
  • Round-robin brainstorming
  • Creative thinking techniques
  • Serendipity technique
  • SCAMPER technique
  • Innovation techniques
  • Value innovation techniques
  • Design thinking techniques
  • Idea generation techniques
  • Personal problems
  • Deciding what career to pursue
  • Managing finances effectively
  • Solving relationship issues
  • Business problems
  • Increasing efficiency and productivity
  • Improving customer service quality
  • Reducing costs and increasing profits
  • Environmental problems
  • Preserving natural resources
  • Reducing air pollution levels
  • Finding sustainable energy sources
  • Individual brainstorming techniques
  • Thinking outside the box
  • Word association and random word generation
  • Mind mapping and listing ideas
  • Group brainstorming techniques
  • Synectics technique
  • Online brainstorming techniques
  • Online whiteboarding tools
  • Virtual brainstorming sessions
  • Collaborative mind mapping software
  • Team activities
  • Group decision making activities
  • Debate activities and role-play scenarios
  • Collaborative problem solving games
  • Creative activities
  • Creative writing exercises and storyboards
  • Imagination activities and brainstorming sessions
  • Visualization activities and drawing exercises
  • Games and puzzles
  • Crossword puzzles and Sudoku
  • Logic puzzles and brain teasers
  • Jigsaw puzzles and mazes
  • Types of decisions
  • Structured decisions
  • Simple decisions
  • Complex decisions
  • Problem solving skills
  • Questioning Assumptions: A Critical Thinking Skill

Understand the importance of questioning assumptions and learn to use it as an effective critical thinking skill when problem solving.

Questioning Assumptions: A Critical Thinking Skill

It's easy to take things at face value and assume that the information we receive is reliable and accurate. But it's important to remember that questioning assumptions is a key critical thinking skill. Taking the time to pause, reflect, and challenge the assumptions we encounter can often lead us to more meaningful insights, solutions, and conclusions. In this article, we’ll explore why it's important to question assumptions, and how to do so in an effective way. By the end of it, you'll have a better understanding of how to think critically and identify potential assumptions in any situation. Questioning assumptions involves looking at the underlying beliefs and ideas that are being used to solve a problem.

This can be done by asking questions such as “What assumptions am I making?” , “What am I assuming is true?” , and “What other possibilities exist?” . By questioning assumptions, we can gain a better understanding of the situation, which can lead to more effective solutions. When questioning assumptions, it is important to keep an open mind and consider all possibilities. This means looking beyond what is immediately obvious and being willing to explore different ideas.

It also involves considering different perspectives and being willing to challenge previously held beliefs. When questioning assumptions, it is also important to consider the context in which the assumptions are being made. This means looking at the environment, culture, values, and beliefs that may be influencing the way we think about a problem. By understanding the context in which assumptions are being made, we can better identify potential biases and blind spots that could be impacting our judgement. In addition to questioning assumptions, it is also important to be aware of how our own biases and values may be influencing our thinking. It is easy to get stuck in our own perspectives and not consider other possibilities.

Using Questioning Assumptions Effectively

In addition, it is important to be aware of how our own biases and values may be influencing our thinking, as well as how our questioning might be influencing others. In order to use questioning assumptions effectively, it is important to be curious and ask questions that are focused on understanding the underlying assumptions and beliefs that drive certain decisions or behaviors. It is also important to recognize that the assumptions we make can shape the way we approach a problem or situation. Asking questions that challenge these assumptions can help us uncover better solutions.

The Benefits of Questioning Assumptions

By taking the time to question assumptions, we can gain a deeper understanding of the context in which a problem is being solved, as well as our own biases and values. This increased understanding can lead to more effective solutions. By questioning assumptions, we can see issues from different angles and gain insight into potential solutions that may not have been apparent before. Additionally, it can help us think outside of the box and challenge conventional wisdom. In addition to problem solving, questioning assumptions can also help us to identify our own personal biases and values. This can be useful when forming opinions or making decisions as it allows us to take a more informed approach.

It also encourages us to think critically and question our own beliefs. Overall, questioning assumptions is an important part of problem solving and critical thinking. It is a process that can help us gain a deeper understanding of the context in which a problem is being solved, as well as our own biases and values. Ultimately, it can lead to more effective solutions. Questioning assumptions is a powerful tool for problem solving and critical thinking. It is a way to uncover hidden assumptions and biases, and look at situations from different angles.

By actively questioning our beliefs, thoughts, and ideas, we can develop better solutions, make better decisions, and create more effective outcomes. Questioning assumptions encourages open-mindedness and encourages us to consider all perspectives. This can ensure that our decision making is based on accurate information and unbiased judgement.

  • information

Virtual Brainstorming Sessions: A Comprehensive Overview

  • Virtual Brainstorming Sessions: A Comprehensive Overview

Learn about the different aspects of virtual brainstorming sessions, including tips for effective online collaboration and brainstorming techniques.

Using Analogies to Solve Problems

  • Using Analogies to Solve Problems

Learn how analogies can help you solve problems more effectively, with advice and examples of different strategies

Cause and Effect Diagrams: A Problem-Solving Technique

  • Cause and Effect Diagrams: A Problem-Solving Technique

A comprehensive guide to understanding and utilizing cause and effect diagrams for problem solving and root cause analysis.

Mind Mapping and Listing Ideas

  • Mind Mapping and Listing Ideas

Explore the individual brainstorming technique of mind mapping and listing ideas to uncover creative solutions.

  • Paired Comparison Analysis: A Comprehensive Overview
  • Finding Sustainable Energy Sources
  • Identifying Patterns: A Practical Guide
  • Creative Writing Exercises and Storyboards
  • Logic Puzzles and Brain Teasers: A Comprehensive Overview
  • Brainwriting: A Group Brainstorming Technique
  • Cost-benefit Analysis: A Guide to Making Informed Decisions
  • Preserving Natural Resources
  • Visualization Activities and Drawing Exercises

Solving Relationship Issues

  • Brainstorming Solutions: A Problem-Solving Guide
  • Maximizing Efficiency and Productivity
  • Jigsaw Puzzles and Mazes: Problem Solving Activities for Fun and Learning
  • Recognizing Bias: A Problem Solving and Critical Thinking Skills Guide
  • Listening Skills: A Comprehensive Overview
  • Exploring Synectics Technique: A Comprehensive Guide
  • Making Complex Decisions: A Comprehensive Overview
  • Round-robin brainstorming: Exploring a Group Brainstorming Technique
  • Managing Your Finances Effectively
  • Simple Decisions - An Overview
  • Exploring Trial and Error Problem Solving Strategies
  • Brainstorming: A Comprehensive Look at Creative Problem Solving
  • Applying Existing Solutions for Problem Solving Strategies
  • Fault Tree Analysis: A Comprehensive Overview
  • Interpreting Information: A Problem-Solving and Research Skills Primer
  • Mind Mapping - Creative Problem Solving and Creative Thinking Techniques
  • Exploring Online Whiteboarding Tools for Brainstorming
  • Brainwriting: A Creative Problem-Solving Technique

Reducing Costs and Increasing Profits: A Problem Solving Example

  • Mind Mapping: A Creative Problem Solving Tool
  • Gantt Charting: A Primer for Problem Solving & Planning Techniques

Exploring the Serendipity Technique of Creative Problem Solving

  • Force Field Analysis for Problem Solving and Decision Making
  • Collaborative Problem Solving Games: Exploring Creative Solutions for Teams
  • Crossword Puzzles and Sudoku: A Problem-Solving Exploration
  • Analyzing Arguments Logically
  • Group Decision Making Activities
  • Exploring the SCAMPER Technique for Creative Problem Solving
  • Choosing the Right Career: Problem-Solving Examples
  • Five Whys Technique: A Comprehensive Analysis
  • Idea Generation Techniques: A Comprehensive Overview
  • Data Collection and Analysis - Problem Solving Skills and Research Skills
  • Negotiation and Compromise
  • Imagination Activities and Brainstorming Sessions
  • Analyzing Consequences: A Problem Solving Strategy
  • Exploring Lateral Thinking: A Comprehensive Guide to Problem Solving Strategies
  • Structured Decisions: An Overview of the Decision Making Process
  • Thinking Outside the Box: An Overview of Individual Brainstorming Techniques
  • Word Association and Random Word Generation
  • How to Explain Ideas Clearly
  • SWOT Analysis: A Comprehensive Overview
  • Improving Customer Service Quality
  • Identifying Root Causes
  • Round-robin Brainstorming: A Creative Problem Solving Tool
  • Design Thinking Techniques: A Comprehensive Overview
  • Collaborative Mind Mapping Software
  • Value Innovation Techniques

New Articles

Solving Relationship Issues

Which cookies do you want to accept?

Logo for The University of Regina OEP Program

Want to create or adapt books like this? Learn more about how Pressbooks supports open publishing practices.

Thinking- question mark

  • Critical thinking (CT) is evaluative.  An evaluation is a statement that compares what is the case to a standard about how things should be. CT requires people to make lots of judgments about good and bad, right and wrong, what we should or shouldn’t do. The standard of evaluation used in critical thinking for reasoning is reliability. Good reasoning is reliable, and bad reasoning is unreliable.
  • In CT, reasoning implies evaluation, both individual (“You should recycle your aluminum!”) and collective (“We should abolish the death penalty!”). Each statement can be supported by reasons, and the reasons can be evaluated as better or worse.
  • In CT, truth is treated as absolute — not partial, changing, or relative to different points of view.
  • The ultimate “should” in critical thinking is this: you should not contradict yourself. There are other “should” statements, but they are all based on this idea that self-contradiction is bad. Contradictory statements, by definition, cannot all be true, and based on #3 above that means they can’t be partly true, or true to some people but not others. A statement that is self-contradictory is absolutely, eternally, necessarily and inevitably not true.

If you don’t agree with one or more of the above assumptions, expect some trouble even understanding what is going on when trying to use critical thinking.  The assumptions listed above are offered not to convince you to accept any of these assumptions or to “prove” them in the formal (CT) sense, but just to offer a bit more about what is assumed in this field.

There is also one more assumption to consider, that is people shouldn’t judge other people’s opinions.

Critical thinking requires energetically judging other people’s opinions (along with our own!) – not in isolation, but in relation to each other. That is, CT requires asking if the reason given to support or back up an opinion is a good one. If no good reason can be found to support an opinion, that opinion is treated as unsupported or unproved. Generally, opinions are better if proved, and not as good if unproved.  By extension, there is a preference for reflective opinions arrived at through slow thinking over opinions of the moment which are formed in fast thinking.

Many people put the majority of their critical thinking energy into judging the thinking of those they disagree with. Our hope is that you will have come to understand that thinking carefully about your own beliefs is worth more of your time, and that you will have come to appreciate the vital importance of people who do not share your same ideas to your process of slow thinking.

Critical Thinking in Academic Research Copyright © 2022 by Cindy Gruwell and Robin Ewing is licensed under a Creative Commons Attribution-ShareAlike 4.0 International License , except where otherwise noted.

Share This Book

Avoid Making Assumptions

What is an assumption in terms of problem solving? It is an idea based on too little or not very good information. For example, the manager of a convenience store has an employee who is often late for her shift. The manager makes the assumption that the employee is lazy and does not take her job seriously. In fact, the employee has had car trouble and must rely on unreliable public transportation to get to work.

When you avoid making assumptions, you get all the information you need before deciding anything. With the right information, you can see the problem clearly rather than focusing on its consequences or mistaking them for the real problem. Then you can work toward a satisfactory solution. For instance, when the manager realizes that transportation is the real problem, she might be able to help the employee find another way to work rather than reprimand her for being lazy .

Continue reading here: Venn Diagram

Was this article helpful?

Related Posts

  • Pretest - Critical Thinking
  • What Is a Judgment Call - Critical Thinking
  • Post Hoc Ergo Propter Hoc
  • Inductive Reasoning - Critical Thinking

Readers' Questions

How does assumption affect possibilities?
Assumptions can greatly impact the possibilities that we consider and ultimately pursue. When we make assumptions, we are limiting our perspective and closing ourselves off to new ideas and opportunities. For example, if someone assumes that they are not capable of learning a new skill, they may never try to learn it and therefore limit their growth and potential. On the other hand, if someone approaches a situation without assumptions and an open mind, they may uncover possibilities that they never even considered before. By challenging our assumptions and being open to new ways of thinking, we can expand our possibilities and create opportunities for growth and success. It is important to remember that assumptions are not always based on reality and can often hold us back from achieving our full potential.
How to avoid embarrassing yourself by making assumptions?
There are several strategies you can follow to avoid embarrassing yourself by making assumptions: Practice active listening: Focus on what others are saying and avoid jumping to conclusions or making assumptions based on partial information. Listen attentively, ask clarifying questions, and seek further information to ensure you have a clear understanding. Seek confirmation: Instead of assuming something to be true, verify your assumptions by asking for confirmation from others. Double-checking facts can help you avoid any potential embarrassment caused by incorrect assumptions. Keep an open mind: Acknowledge that your assumptions might not always be accurate and be willing to challenge them. Embrace a mindset of curiosity and be open to new information or perspectives that may contradict your assumptions. Consider different perspectives: Try to put yourself in others' shoes and consider alternative viewpoints. Recognize that different people have different experiences and beliefs, which may affect their perspectives and opinions. Avoid assuming that your own perspective is always correct. Ask questions: When in doubt, ask questions to gather more information and clarify any uncertainties. This can help you avoid faulty assumptions and foster a better understanding of the situation. Reflect on past experiences: If you have made embarrassing assumptions in the past, take some time to reflect on those situations. Analyze what led you to make those assumptions and identify any patterns or triggers. Learning from past mistakes can help you avoid making similar assumptions in the future. Cultivate self-awareness: Be mindful of your own assumptions and biases. Recognize that everyone has biases, and be proactive in challenging your own assumptions. This self-awareness will help you become more conscious of your thinking patterns and prevent embarrassment caused by faulty assumptions. Remember, it's impossible to completely eliminate all assumptions. However, by consciously practicing these strategies and cultivating a mindset of curiosity and open-mindedness, you can significantly minimize the risk of embarrassing yourself through assumptions.
How to avoid assumptions in critical thinking?
To avoid assumptions in critical thinking, you can follow these strategies: Be aware of your biases: Recognize that you have inherent biases and preconceived notions that can influence your thinking. Try to be conscious of these biases and actively challenge them. Seek multiple perspectives: Consider alternative viewpoints and opinions before forming your own conclusions. Listen to different arguments and weigh the evidence objectively. Question everything: Develop a habit of questioning information, claims, and assumptions. Ask yourself: What evidence supports this claim? Are there any counterarguments? Is this information reliable and accurate? Gather evidence: Base your conclusions on verifiable evidence rather than assumptions or personal beliefs. Seek credible sources, conduct research, and critically evaluate the information before drawing any conclusions. Avoid generalizations: Avoid making sweeping generalizations based on limited evidence or personal experiences. Recognize that each situation is unique, and making general assumptions can lead to inaccurate conclusions. Consider probability: Instead of assuming something to be true without evidence, assess the likelihood of it being true based on the available evidence. Avoid jumping to conclusions without proper reasoning. Practice empathy: Put yourself in others' shoes and try to understand their perspectives. Be open to different viewpoints and consider how biases and assumptions may affect your understanding of a particular issue. Be open to change: Be willing to revise your beliefs and opinions based on new information and evidence. Avoid rigid thinking and embrace intellectual flexibility. Validate information: Verify the accuracy and reliability of information by cross-referencing multiple sources. Avoid relying on single sources or unverified claims. Reflect and evaluate: Continuously reflect on your own thinking processes, assumptions, and biases. Regularly evaluate your critical thinking skills and identify areas for improvement. Remember that avoiding assumptions is not about eliminating all assumptions entirely, but rather being aware of them and examining them critically.
How to make critical ssumptions?
To make critical assumptions, follow these steps: Identify the purpose and scope: Clearly define the problem or situation you are working on and identify the objective you want to achieve. Determine the extent of knowledge required to make assumptions. Gather relevant information: Collect all the available data, information, and evidence related to the problem or situation. Ensure that the sources are reliable and accurate. Analyze the information: Carefully examine and evaluate the information you have gathered. Look for patterns, discrepancies, or gaps that may affect your assumptions. Identify potential biases: Be aware of your own biases and any potential biases in the information you have collected. Consider the source of the information and any possible vested interests that may affect its reliability. Consider multiple perspectives: Evaluate the problem or situation from different angles and viewpoints. This helps you to see different possibilities and anticipate potential outcomes. Apply critical thinking: Engage in thorough and objective analysis. Ask probing questions, challenge assumptions, and consider alternative explanations or scenarios. Make assumptions based on evidence: Base your assumptions on logical reasoning and evidence from the information you have gathered. Avoid assumptions that are solely based on personal beliefs or opinions. Test assumptions: Validate your assumptions by checking them against additional evidence or by conducting experiments or tests. This ensures that your assumptions are reliable and accurate. Revise assumptions if necessary: Be open to revising your assumptions if new information or evidence emerges that contradicts or challenges your initial assumptions. Communicate assumptions: Clearly communicate your assumptions to others involved in the decision-making process or affected by the situation. This facilitates transparency and ensures that assumptions are understood by all parties. Remember, critical assumptions are based on thorough analysis, evidence, and a willingness to revise or adjust them as new information becomes available.
How to apply critical thinking techniques and methods(avoiding assumptions?
Question assumptions: Challenge the underlying assumptions behind an argument or statement. Ask yourself if there is any evidence to support these assumptions or if they are based on biases or conjecture. Seek clarity: Clarify vague or unclear statements by asking for more information or definitions. This helps prevent jumping to conclusions based on incomplete or misunderstood information. Consider alternative perspectives: Look at the issue from different angles and consider conflicting viewpoints. This allows you to evaluate the strengths and weaknesses of each perspective and avoid forming biased or one-sided opinions. Evaluate evidence: Examine and analyze the evidence or information presented to support an argument. Assess the credibility and reliability of the sources, as well as the sufficiency and relevance of the evidence. Avoid accepting information at face value without verifying its accuracy. Identify and analyze reasoning: Pay attention to the reasoning used to support arguments or claims. Look for logical fallacies or flawed reasoning, such as generalizations, false cause-effect relationships, or appeal to emotions. Assess the strength and validity of the reasoning presented. Use logical thinking: Apply logical reasoning to evaluate arguments. Assess if the premises logically lead to the conclusion. Use deductive and inductive reasoning to identify any weaknesses in the argument's logic. Stay open-minded: Be receptive to new information and revise your beliefs or opinions based on evidence. Avoid being overly attached to your own ideas, as this can hinder critical thinking. Avoid biases: Be aware of your own biases and consciously try to eliminate them when analyzing information or making judgments. Biases can cloud your judgment and prevent objective analysis. Use reflective thinking: Take the time to reflect on your own thinking process. Analyze how you arrived at a certain conclusion and if there are any gaps or biases in your own reasoning. Reflective thinking helps strengthen critical thinking skills and improve decision-making. Engage in active learning: Continuously learn and seek out new knowledge. Regularly engage in activities that challenge your thinking and improve your critical thinking skills, such as reading challenging books or engaging in thought-provoking discussions.
How avoiding assumptions in workplace helps develop critical thinking skills?
Avoiding assumptions in the workplace can help develop critical thinking skills in several ways: Increases objectivity: By avoiding assumptions, individuals are forced to look at situations objectively and consider all available evidence. This helps develop critical thinking skills by encouraging individuals to analyze information in a logical and unbiased manner. Encourages open-mindedness: Avoiding assumptions requires individuals to withhold judgment until they have gathered enough information and considered different perspectives. This fosters open-mindedness, which is a key aspect of critical thinking. It allows individuals to explore various possibilities and consider different solutions to problems. Promotes better decision-making: When assumptions are avoided, individuals are less likely to make hasty decisions based on limited or incomplete information. Critical thinking involves analyzing the situation from multiple angles, gathering evidence, and considering different alternatives. This approach to decision-making helps individuals make more informed and thoughtful choices. Enhances problem-solving abilities: Avoiding assumptions helps individuals identify and challenge their biases and preconceived notions. This is crucial for effective problem-solving. By considering alternative viewpoints and potential solutions, individuals can develop more creative and innovative approaches to solving workplace problems. Facilitates effective communication: Assumptions often lead to misunderstandings and miscommunication in the workplace. By avoiding assumptions and actively listening to others, individuals can better understand different perspectives and engage in more productive discussions. This enhances critical thinking skills as it requires individuals to consider multiple viewpoints and integrate them into their own thinking. Overall, avoiding assumptions in the workplace promotes critical thinking by fostering objectivity, open-mindedness, better decision-making, enhanced problem-solving abilities, and effective communication. These skills are valuable for individual growth, team collaboration, and overall organizational success.
How does avoiding assumption contribute to critical thinking techniques and methods?
Avoiding assumptions is a crucial aspect of critical thinking as it helps in effectively arriving at well-reasoned and logical conclusions. Here are some ways in which avoiding assumptions contributes to critical thinking techniques and methods: Enhanced accuracy: When assumptions are made without sufficient evidence or reasoning, it can lead to flawed conclusions. By avoiding assumptions, critical thinkers are more likely to gather accurate and reliable information, ensuring that their reasoning and analysis are based on solid foundations. Objective analysis: Assumptions tend to be biased or influenced by personal beliefs or preconceived notions. Avoiding assumptions allows critical thinkers to approach a situation or problem objectively. They consider multiple perspectives, assess evidence impartially, and remain open to new information, thus fostering a more well-rounded analysis. Identification of biases: Assumptions often result from biases, stereotypes, or generalizations. By actively avoiding assumptions, critical thinkers can pinpoint their own biases and challenge them. This helps in promoting a fair and unbiased evaluation of information, leading to a deeper understanding of complex issues. Improved problem-solving: Assumptions can limit one's ability to think creatively and find innovative solutions. By avoiding assumptions, critical thinkers are encouraged to explore various possibilities, question traditional wisdom, and consider alternative viewpoints. This broader mindset can lead to more effective problem-solving techniques. Increased intellectual humility: Avoiding assumptions requires consistently questioning and challenging one's own beliefs and ideas. This process fosters intellectual humility by recognizing the limitations of one's knowledge and being open to the idea that others may have valid perspectives. Critical thinkers who avoid assumptions are more likely to embrace lifelong learning and seek continuous improvement. Effective communication: Assumptions can create misunderstandings and misinterpretations in communication. By avoiding assumptions, critical thinkers are more inclined to actively listen, clarify information, and ask relevant questions. This promotes clearer and more accurate communication, reducing the chance of errors or miscommunication. Overall, avoiding assumptions is a fundamental component of critical thinking, enabling individuals to think critically, analyze information objectively, and arrive at well-informed conclusions.
How does assumption hinder critical thinking?
Assumptions can hinder critical thinking because they are often based on incomplete or inaccurate information and can lead to flawed reasoning. When individuals rely on assumptions without questioning their validity or seeking evidence to support them, they may overlook alternative perspectives and potential solutions. This can prevent them from fully analyzing a situation or problem and coming up with the most effective and accurate conclusions. Assumptions can also create biases and preconceived notions that cloud judgment and prevent open-minded thinking. Overall, critical thinking requires careful examination of assumptions and a willingness to challenge them in order to arrive at well-informed and logical conclusions.
What is assumption in critical thinking?
An assumption in critical thinking refers to a belief or a statement that is taken for granted or accepted as true without proper evidence or justification. In the context of critical thinking, assumptions can be both implicit (not explicitly stated) or explicit (clearly stated). In order to think critically, it is important to identify and evaluate assumptions, as they can impact the validity and soundness of an argument or a proposition.
Do good critical thinkers avoid making working assumptions?
Good critical thinkers do not avoid making working assumptions entirely, as assumptions can be useful in the thinking process. However, they are careful about the assumptions they make and approach them with skepticism, subjecting them to critical analysis and evaluation. A working assumption is a provisional assumption made in the absence of complete or definitive information. It helps to guide thinking and decision-making when faced with incomplete data or uncertainty. Critical thinkers recognize that assumptions can be necessary and useful in the absence of complete information, but they also understand the limitations and potential biases associated with making assumptions. To effectively use working assumptions, critical thinkers: Acknowledge assumptions: They are aware of and acknowledge the assumptions they are making. They do not take assumptions for granted or see them as absolute truth. Subject assumptions to critical evaluation: Critical thinkers question and evaluate assumptions rigorously, seeking evidence and logical reasoning to support or challenge them. They validate or modify assumptions based on available information. Recognize biases: They are aware of their own biases and biases that may be present in assumptions. They strive to minimize the impact of biases in their thinking process. Seek alternative perspectives: Critical thinkers actively seek out alternative perspectives and consider multiple assumptions that may be applicable to a given situation. They recognize that different assumptions can lead to different conclusions and outcomes. Revisit assumptions: As new information becomes available, critical thinkers reassess their assumptions to ensure they align with the latest knowledge and understanding. In summary, good critical thinkers do not avoid making working assumptions. Instead, they approach assumptions with skepticism, critically evaluate them, and remain open to revising or discarding them as new information emerges.

Watson-Glaser Assessments: Guidelines & Practice Examples

Originally designed by Goodwin Watson and Edward Glaser, The Watson-Glaser Critical Thinking Appraisal (Form AM) was a measurement of “how well you are able to reason analytically and logically.” After their extensive use during World War One, experts increasingly used the tests as a tool to rank and filter individuals in contexts including (but not limited to) education and employment. In this article, we’ll be providing you with an overview of The Watson-Glaser Critical Thinking Appraisal, its test format, and question examples for your practice.

Table of Contents

What is a Watson Glaser test?

The Watson Glaser Assessment (Watson Glaser Critical Thinking Appraisal) is designed by Pearson Talentlens – a part of the famous Pearson education publishing house. The assessment is a quick, consistent, and accurate measurement of the test-takers ability to analyze, reason, interpret and draw logical conclusions from written information .

This critical thinking test has five scales which are the elementary units of critical thinking and reasoning . These five units serve as parameters to measure all areas of critical thinking ability.

The test is administered to appraising adults (16 years and above) with questions of varying difficulty and format.

Watson Glaser Test format

The Watson Glaser Critical Thinking Appraisal is a timed test. It can be administered both online and offline, depending on the company you are applying to.

  • For the online version (W-G III), the test is suitable for the unproctored screening of job applicants . Questions are selected randomly from a larger pool of questions called “item bank”. This helps minimize the possibility of cheating due to the fact that there are no applicants having the same set of questions.
  • For the offline version (W-G II Forms D and E), applicants have to take the test under supervised conditions .

Upon finishing, applicants receive a percentile score with norm groups either provided by Pearson Talentlens or designed by the company.

Popular test formats are as follows:

  • 40 questions – 30 minutes
  • 80 questions – 60 minutes

The question distribution in a 40-question Watson Glaser test includes the following:

  • Inference – 5 questions
  • Recognition of assumptions – 12 questions
  • Deduction – 5 questions
  • Interpretation – 6 questions
  • Evaluation – 12 questions

Watson Glaser test: 5 question types & examples

Five types of questions appearing in the test are

Infer questions

  • Recognize Assumption questions

Deduce questions

Interpret questions.

  • Evaluate Argument questions

critical thinking assumption questions

You will find five choices of answers for each inference in the question: True, Probably True, Insufficient Data, Probably False, and False. Your job is to determine which choice best fits the hypothesis.

critical thinking assumption questions

  • True : If you think the inference is definitely TRUE; that it properly follows beyond a reasonable doubt from the statement of facts given.
  • Probably True : If, in the light of the facts given, you think the inference is Probably True; that it is more likely to be true than false.
  • Insufficient Data : If you decide that there are Insufficient Data; that you cannot tell from the facts given whether the inference is likely to be true or false; if the facts provide no basis for judging one way or another.
  • Probably False : If, in the light of the facts given, you think the inference is Probably False; that it is more likely to be true than false.
  • False : If you think the inference is definitely False; that it is wrong, either because it misinterprets the facts given, or because it contradicts the facts or necessary inferences from those facts.

Unlike popular aptitude tests where you use solely given information, this test allows the use of specific commonly accepted knowledge or information that practically every person has.

critical thinking assumption questions

Source: Pearson Talentlens

During the past month, managers scheduled for international assignments voluntarily attended our company’s cross-cultural business training workshop. All of the managers reported that the quality of the training was high and focused on valuable work skills that could be immediately applied.

The majority of training was devoted to rules and regulations for doing business in this country. 

A. True B. Probably True C. Insufficient Data D. Probably False E. False

Answer : Probably False

Explanation : Probably False because the training focused on cross-cultural business. It cannot be considered definitely false because the specific course content is not provided.

Practice Example:

Chamonix is one of the oldest ski resorts in France. Last year, the Chamonix-Mont-Blanc Valley authorities introduced a climate and energy action plan, the first in the French Alps. The plan commits to reducing greenhouse gas emissions in the area by 22 percent. Among the proposed measures is a ban on the most polluting lorries using the nearby Mont Blanc tunnel. Climate change will have a major impact on the valley’s main economic activities: less snow on low-altitude ski slopes and the risk of increased pressure on high-altitude ski slopes have been recorded in recent studies. Natural habitats, river patterns, forests, and agriculture might be radically transformed, increasing the likelihood of hazards such as avalanches, floods, and landslides.

Although air quality is a great concern for those living at the foot of Mont Blanc, noise pollution is arguably the most noticeable at a local level.

A. True B. ProbablyTrue C. Insufficient Data D. Probably False E. False

Answer : Insufficient Data

Explanation : The passage does not mention noise pollution, as well as its existence at local level.

Recognize assumption questions

critical thinking assumption questions

In this question, you are presented with a statement and two choices “ Assumption made ” or “ Assumption not made “. Your job is to determine whether the statement is an assumption taken for granted (Assumption made) or an assumption not necessarily taken for granted (Assumption not made).

critical thinking assumption questions

We need to save time in getting there so we’d better go by plane.

There is a plane service available to us for at least part of the distance to the destination. 

A. Assumption made B. Assumption not made

Answer : Assumption made

Explanation :

This is assumed in the statement because, in order to save time by plane, it must be possible to go by plane.

While owning a pet can be expensive and is occasionally an inconvenience, it’s a good thing to do if you want to improve your chances of living a satisfying life.

Pet owners will always lead more satisfying lives than those who don’t own pets.

Answer : Assumption not made

From the given premises, we can only see that: if you want a satisfying life, it is a good thing to have a pet. However, having a pet does not definitely lead to a satisfying life. There is no indication to compare the satisfaction between having a pet or not having a pet as a way to live a satisfying life. 

critical thinking assumption questions

In this type of question, each item contains several statements (premises) followed by several suggested conclusions. For the purposes of the test, you must consider the given statements as true without exception. The question presents you with two options

  • Conclusion follows : If you think the conclusion necessarily follows from the statements given;
  • Conclusion does not follow : If you think it is not a necessary conclusion from the statements given. 

Since the test requires you to regard given statements as completely true, you have to try not to let your common sense and existing knowledge interfere. You have to stick to only the given statements (premises) and make judgments as to whether it necessarily follows from the statement or not.

One thing to note is the use of the word “Some” in this type of question. It means an indefinite part or quantity of a class of things. It can be either a portion or perhaps all of the class.

critical thinking assumption questions

Sample: 

Some Sundays are rainy. All rainy days are boring. Therefore …

Some Sundays are boring.

A. Conclusion follows B. Conclusion does not follow

Answer : Conclusion follows

The conclusion necessarily follows from the statements because, according to them, rainy Sundays must be boring.

Damage to roads in the area has made them unsuitable for loads over one ton. These loads are being transported to the capital by air, but as air resources are limited they are restricted to carrying food and medical supplies. Roads cannot be repaired until medical emergencies are dealt with. Therefore, …

Food can be taken to the capital by road.

Answer : Conclusion does not follow

Roads can not be used to transport food in loads over one ton. Food loads over one ton are being transported by air. We are not sure whether food in loads under one ton are delivered by road or anything else.

critical thinking assumption questions

An interpret question provides you with a short paragraph followed by several suggested conclusions. You must assume that everything in the short paragraph is true, for the purpose of the test. To answer the question, you have to judge whether or not each of the suggested conclusions logically follows beyond a reasonable doubt from the information presented in the paragraph.

You have two answer choices:

  • Conclusion follows : If you think the conclusions follow beyond a reasonable doubt (although they may not follow absolutely and necessarily);
  • Conclusion does not follow : If you think the conclusions doesn’t follow beyond a reasonable doubt from the facts given in the short paragraph. 

As a rule of thumb, you should judge each conclusion independently from your common sense or outside knowledge.

critical thinking assumption questions

Source: Watson Glaser

A study of vocabulary growth in children from ages eight months to six years old shows that the size of spoken vocabulary increases from zero words at age eight months to 2562 words at age six years.

Vocabulary is slowest during the period when children are learning to walk.

Answer : Conclusion does not follow.

The conclusion does not follow because there is no information given that relates the growth of vocabulary to walking.

An accounting computer program, MagicNumber, is Wisdom Software’s biggest-selling product, with its development involving 20% of programmers and 30% of marketing staff. DesignAid, a graphic design program, is the latest offering from the company. It is definitely expected to sell more copies than MagicNumber and will have fewer programmers working on its design, but more marketing staff.

DesignAid will bring in greater profits for the company than MagicNumber.

The paragraph only mentioned that DesignAid is expected to sell more than MagicNumber. This does not necessarily mean that DesignAid uiwll bring greater profit than MagicNumber.

Evaluate argument questions

critical thinking assumption questions

The question involves distinguishing between strong and weak arguments, as far as the question at issue is concerned. In each question, there is a series of arguments that you must regard as true. Your job is to determine whether or not each of these arguments is a strong or a weak one.

  • Argument strong : If you think the argument is strong;
  • Argument weak: If you think the argument is weak.

For an objective evaluation of the argument, you must judge each argument independently on its own merit, without the influence of your personal perception.

One thing to note is the use of the word Should. By using Should at the beginning of each question, it means “Would the proposed action promote the general welfare of the people in your country?”

critical thinking assumption questions

Should young adults in this country go to university?

No; a large percentage of young adults do not have enough ability or interest to derive any benefit from university training.

A. Argument strong B. Argument weak

Answer : Argument strong.

If this is true, as the directions require us to assume, it is a weighty argument against all young adults going to university.

Practice Example 5:

Is it worthwhile for a business to invest in training employees?

Yes, research shows the amount of money spent on training is positively related to profitability.

A. Argument Strong B. Argument Weak

Answer : Argument Strong

The explanation show that business can increase their profit by invest more money on training. This is a direct back up evidence for the claim.

Which companies use Watson Glaser tests?

Watson Glaser Tests are popular in the pre-employment process across sectors such as medical, marketing, education, legal, and professional services. The test can be used for different job levels like Graduates, Execs and Managerial, Supervisors, and Professionals., depending on the company hiring request.

There are a great number of UK companies using it for their pre-employment screening process. The names include the following

  • Simmons & Simmons
  • Hill Dickinson
  • Bank of England
  • Burges Salmon
  • Ince & Co
  • Government Legal Service
  • Irwin Mitchell
  • Clifford Chance
  • Hogan Lovells

Prepare for aptitude tests with MConsultingPrep

Explore MConsultingPrep’s Aptitude Test – a strategic & comprehensive practicing platform with

  • A question bank of +1000 aptitude questions (Numerical, Verbal & Logical)
  • A comprehensive explanation of every question
  • Evaluation and recording tools for performance reviews

You’ll be practicing more strategically with our test packages, to gain more confidence for every test question you might experience in your future aptitude tests.

Our massive question bank consists of a significant number of question variations from many test publishers and companies, across different companies & businesses.

With questions organized by type, you can leverage your preparation by

  • Focusing on practicing questions in specific types
  • Improving your performance in questions that you struggle with

With evaluation tools coming in handy, you have better insights into your overall performance, what you excel at, and what you need improvements on.

Scoring in the McKinsey PSG/Digital Assessment

The scoring mechanism in the McKinsey Digital Assessment

Related product

Thumbnail of Aptitude Test Package

Aptitude Test Package

Simulating most common test publishers, this package provides you with 1400+ numerical, verbal and logical reasoning questions. Ace the aptitude test with our practical study guides tailored to each question type.

Let’s dive deep into aptitude tests: definition, different types, and free practice materials for this world-famous assessment tool!

Aptitude tests are usually compulsory in the application process. So how much time should you spend on learning to get a high score?

While the majority of aptitude tests contain multiple-choice questions, some test providers provide gamified assessments. Dive in the details now!

  • Practice Tests
  • Predictive Index
  • Firefighter
  • Hogan Assessments
  • Leadership Assessment
  • Ramsay Technician Assessments
  • Watson-Glaser
  • Raven's Progressive Matrix
  • NEO Personality Inventory
  • Texas Success Initiative
  • Birkman Personality Test
  • TSA Prep Booster™ Course
  • TSA Practice Test
  • TSA Written Skills Assessment
  • TSA CBT X-Ray Object Recognition Test
  • TSA Connect the Dots
  • SHL Assessment Prep Course
  • Practice Test & Answers
  • SHL Practice Tests
  • SHL Test Answers
  • SHL Inductive Reasoning Test
  • SHL Numerical Reasoning Test
  • SHL Verbal Reasoning Test
  • SHL Verify G+ Test
  • SHL Mechanical Comprehension Test
  • SHL Situational Judgment Test
  • SHL OPQ Personality Test
  • Predictive Index Master (Cognitive & Behavioral)
  • Predictive Index Cognitive Assessment
  • Predictive Index Behavioral Assessment
  • Predictive Index Practice Test
  • Predictive Index Results
  • Caliper Course
  • Caliper Test Prep With Real Practice Test
  • USPS Postal Exam
  • Postal Exam 474
  • Postal Exam 475
  • Postal Exam 476
  • Postal Exam 477
  • USPS Postal Exam Prep
  • Pass the 2024 Postal Exam With Practice Tests
  • Virtual Entry Assessment (VEA)
  • General Police Prep Course
  • Police Situational Judgement Test
  • Police Psychological Exam Course
  • Massachusetts State Police Exam
  • Pennsylvania Police Exam
  • Philadelphia Police Exam
  • Nassau County Police Exam Course
  • Suffolk County Police Exam
  • Correctional Officer Exam
  • MTA Police Exam
  • New York State Police Exam Prep Course
  • School Safety Agent Course
  • Police Officer NYPD Exam
  • Police Fitness Prep Course
  • Exam Formats
  • EB Jacobs Law Enforcement Aptitude Battery
  • CJBAT Study Guide
  • DELPOE Police Exam
  • Texas LEVEL Test With Expert Guides
  • PELLETB Course
  • FBI Test Phase 1 (Special Agent Exam): Guide with Practice Test [2024]
  • Police Test Preparation Suite
  • Pass a Polygraph Test (Lie Detector): Expert Tips & Questions – 2024
  • Firefighter Test
  • FDNY Firefighter Prep Course
  • Firefighter Psych Test
  • NFSI Firefighter Prep Course
  • FCTC Firefighter Prep Course
  • Firefighter Aptitude and Character Test
  • FireTeam Prep Course
  • Master Course
  • Hogan Assessments Master Course
  • Personality Courses
  • Hogan Personality Inventory (HPI)
  • Hogan Development Survey (HDS)
  • Hogan Motives, Values & Preferences Inventory (MVPI)
  • Busines Reasoning Course
  • Hogan Business Reasoning Inventory (HBRI)
  • Leadership Assessment Test
  • GardaWorld Pre Board Primer
  • Bennett Mechanical Comprehension Test II (BMCT-II) Success Prep Course
  • Beat the 2024 BMCT With Industry Expert Guides & Realistic Practice Tests
  • 911 Dispatcher
  • CHP Dispatcher
  • Exam Format
  • Criticall Dispatcher
  • Criticall Dispatcher Test
  • Criteria Cognitive Aptitude Test - CCAT Course
  • Universal Cognitive Aptitude Test - UCAT Course
  • CCAT Practice Test
  • Criteria Pre-employment Testing: Personality, Aptitude & Skill Tests
  • Korn Ferry Course
  • Ace the 2024 Korn Ferry Assessment With Practice Test & Expert Guides
  • Ramsay Electrical Assessment
  • Ramsay Maintenance Assessment
  • Ramsay Mechanical Assessment
  • Ramsay Multicraft Assessment
  • Ramsay Electrical Practice Test
  • Ramsay Maintenance Practice Test
  • Ramsay Mechanical Practice Test
  • Ramsay Multicraft Practice Test
  • Ramsay Test Prep
  • AFOQT Study Guide
  • ASTB Study Guide
  • SIFT Study Guide
  • Watson-Glaser Critical Thinking Course
  • Beat the Watson Glaser and Upgrade Your Career
  • Raven's Advanced Progressive Matrices
  • Texas Success Initiative Course
  • TSI Practice Test 2024: Math, Reading & Writing
  • TSI Reading Practice Test: 15 Q&A with Explanations
  • Pass our Free TSI Math Practice Test (2024 Update)
  • Take our Free TSI Writing Practice Test (2024)
  • Birkman Personality Course
  • How it Works

Critical Thinking Test: Sample Questions with Explanations (2024)

Employers value and seek candidates who demonstrate advanced critical thinking skills. They often administer critical thinking tests as part of their hiring process. Critical thinking tests can be very difficult for those who don’t prepare. A great way to start practicing is by taking our critical thinking free practice test.

What Does The Critical Thinking Test Include?

The Critical Thinking Test assesses your capacity to think critically and form logical conclusions when given written information. Critical thinking tests are generally used in job recruitment processes, in the legal sector. These tests measure the analytical critical thinking abilities of a candidate.

Why Is Critical Thinking Useful?

Critical thinking is put into action in various stages of decision-making and problem-solving tasks:

  • Identify the problem
  • Choose suitable information to find the solution
  • Identify the assumptions that are implied and written in the text
  • Form hypotheses and choose the most suitable and credible answers
  • Form well-founded conclusions and determine the soundness of inferences

What is Watson Glaser Test and what Critical Thinking Skills it Measures?

The most common type of critical thinking test is the Watson-Glaser Critical Thinking Appraisal (W-GCTA). Typically used by legal and financial organizations, as well as management businesses, a Watson Glaser test is created to assess candidates’ critical thinking skills.

The test consists of 10 questions to be answered in 10 minutes approx (although there is no timer on the test itself). Our test is slightly harder than the real thing, to make it sufficiently challenging practice.

You need to get 70% correct to pass the test. Don’t forget to first check out the test techniques section further down this page beforehand.

Questions          25

Pass percentage          70%.

The test is broken down into five central areas:

  • Assumptions
  • Interpretation

Critical Thinking Course

  • 1 BONUS Interview Prep Video Guide Buy this Course: Get full access to all lessons, practice tests and guides.

The Five Critical Thinking Skills Explained

1. recognition of assumption.

You’ll be presented with a statement. The statement is then followed by several proposed assumptions. When answering, you must work out if an assumption was made or if an assumption was not made in the statement. An assumption is a proclamation that an individual takes for granted. This section of the tests measures your ability to withhold from forming assumptions about things that are not necessarily correct.

  • 1: Assumption Made
  • 2: Assumption Not Made

Although the passage does state that Charlie’s fundraising team is doing its best so that the charity event can meet its goal, nowhere did it state that their team is leading the event.

2. Evaluation of Arguments

You will be presented with an argument. You will then be asked to decide whether the argument is strong or weak. An argument is considered strong if it directly connects to the statement provided, and is believed to be significant.

No, participation awards should not be given in every competition because studies have shown that this would cause the participants to put in less effort because they will get a prize no matter what the outcome is.

  • 1: Strong Argument
  • 2: Weak Argument

This is a strong argument as it provides evidence as to why participation awards should not be given in every competition

3. Deductions

In deduction questions, you will need to form conclusions based solely on the information provided in the question and not based on your knowledge. You will be given a small passage of information and you will need to evaluate a list of deductions made based on that passage. If the conclusion cannot be formed for the information provided, then the conclusion does not follow. The answer must be entirely founded on the statements made and not on conclusions drawn from your knowledge.

In a surprise party for Donna, Edna arrived after Felix and Gary did. Kelly arrived before Felix and Gary did.

  • 1: Conclusion Follows
  • 2: Conclusion Does not Follow

For questions like this, jot down the clues to help you out. Use initials as a quick reference.

K | F&G | E

Looking at the simple diagram, “K”, which stands for “Kelly,” arrived before Edna “E” did. The answer is A.

4. Interpretation

In these questions, you are given a passage of information followed by a list of possible conclusions. You will need to interpret the information in the paragraph and determine whether or not each conclusion follows, based solely on the information given.

A number of students were given the following advice:

“The use of powerful words is a technique, which makes you a better writer. Your choice of words is very important in molding the way people interaction with the article. You should use powerful words to spice up your article. Power words should be used liberally to enhance the flavor of what you write! ”

In the fourth sentence, it is stated, “Power words should be used liberally to enhance the flavor of what you write!”

Thus, if you were to write an essay, using powerful words can give more flavor to it.

5. Inferences

An inference is a conclusion made from observed or supposed facts and details. It is information that is not apparent in the information provided but rather is extracted from it. In this section, you will be provided with a passage of information about a specific scene or event. A list of possible inferences will then be given, and you will need to decide if they are ‘true’, ‘false’, ‘possibly true’, ‘possibly false’, or whether it is not possible to say based on the information provided.

With the advancement of technology, the need for more infrastructure has never been higher. According to the plan of the current U.S. Administration, it aims to put a $1 trillion investment on improving infrastructure, a portion of which will include priority projects and technologies that can strengthen its economic competitiveness such as transportation, 5G wireless communication technology, rural broadband technologies, advanced manufacturing technologies, and even artificial intelligence.

It stated that it expects to work with Congress to develop a comprehensive infrastructure package, which is expected to have a budget of $200 billion for certain priorities.

  • 2: Probably True
  • 3: Not Enough Information
  • 4: Probably False

Although it was mentioned in the passage that the U.S. government is to allocate $200 billion on certain priorities, it did not specify if these certain priorities were for ‘transportation, 5G wireless communication technology, rural broadband technologies, advanced manufacturing technologies, and artificial intelligence’ or if the aforementioned priorities will have a different allocation.

What we can be sure of, however, is that at least a portion of the $1 trillion infrastructure budget will be used on the mentioned priorities regardless, meaning that there is a chance that $200 billion will be used on those aforementioned areas.

Improve Your Score with Prepterminal’s Critical Thinking Course

The Critical Thinking test is difficult, but not impossible to overcome with practice. At PrepTerminal our psychometric test experts have developed a critical thinking preparatory test to provide you with the material you need to practice for your critical thinking test. Prepare with us to increase your chance of successfully overcoming this hurdle in the recruitment process.

Prepterminal’s preparatory critical thinking course features a structured study course along with critical thinking practice tests to help you improve your exam score. Our course includes video and text-based information presented in a clear and easy-to-understand manner so you can follow along at your own pace with ease.

Matt

Created by: Matt

Psychometric tutor, prepterminal test expert, 414 students, 4.7 , 73 reviews.

Bookmark this page

Translate this page from English...

*Machine translated pages not guaranteed for accuracy. Click Here for our professional translations.

Critical Thinking: Basic Questions & Answers




In this interview for Think magazine (April ’’92), Richard Paul provides a quick overview of critical thinking and the issues surrounding it: defining it, common mistakes in assessing it, its relation to communication skills, self-esteem, collaborative learning, motivation, curiosity, job skills for the future, national standards, and assessment strategies.

Critical thinking is essential to effective learning and productive living. Would you share your definition of critical thinking?

First, since critical thinking can be defined in a number of different ways consistent with each other, we should not put a lot of weight on any one definition. Definitions are at best scaffolding for the mind. With this qualification in mind, here is a bit of scaffolding: critical thinking is thinking about your thinking while you’re thinking in order to make your thinking better. Two things are crucial:

critical thinking is not just thinking, but thinking which entails self-improvement

this improvement comes from skill in using standards by which one appropriately assesses thinking. To put it briefly, it is self-improvement (in thinking) through standards (that assess thinking).

 

Could you give me an example?

Certainly, one of the most important distinctions that teachers need to routinely make, and which takes disciplined thinking to make, is that between reasoning and subjective reaction.

")

), systematically misleading the 150,000 or so teachers who read the publication.

Could this possibly be a rare mistake, not representative of teacher knowledge?

I don't think so. Let me suggest a way in which you could begin to test my contention. If you are familiar with any thinking skills programs, ask someone knowledgeable about it the "Where's the beef?" question. Namely, "What intellectual standards does the program articulate and teach?" I think you will first find that the person is puzzled about what you mean. And then when you explain what you mean, I think you will find that the person is not able to articulate any such standards. Thinking skills programs without intellectual standards are tailor-made for mis-instruction. For example, one of the major programs asks teachers to encourage students to make inferences and use analogies, but is silent about how to teach students to assess the inferences they make and the strengths and weaknesses of the analogies they use. This misses the point. The idea is not to help students to make more inferences but to make sound ones, not to help students to come up with more analogies but with more useful and insightful ones.

What is the solution to this problem? How, as a practical matter, can we solve it?

Well, not with more gimmicks or quick fixes. Not with more fluff for teachers. Only with quality long-term staff development that helps the teachers, over an extended period of time, over years not months, to work on their own thinking and come to terms with what intellectual standards are, why they are essential, and how to teach for them. The State Department in Hawaii has just such a long-term, quality, critical thinking program (see " "). So that's one model your readers might look at. In addition, the National Council for Excellence in Critical Thinking Instruction is focused precisely on the articulation of standards for thinking. I am hopeful that eventually, through efforts such as these, we can move from the superficial to the substantial in fostering quality student thinking. The present level of instruction for thinking is very low indeed.

But there are many areas of concern in instruction, not just one, not just critical thinking, but communication skills, problem solving, creative thinking, collaborative learning, self-esteem, and so forth. How are districts to deal with the full array of needs? How are they to do all of these rather than simply one, no matter how important that one may be?

This is the key. Everything essential to education supports everything else essential to education. It is only when good things in education are viewed superficially and wrongly that they seem disconnected, a bunch of separate goals, a conglomeration of separate problems, like so many bee-bees in a bag. In fact, any well-conceived program in critical thinking requires the integration of all of the skills and abilities you mentioned above. Hence, critical thinking is not a set of skills separable from excellence in communication, problem solving, creative thinking, or collaborative learning, nor is it indifferent to one's sense of self-worth.

Could you explain briefly why this is so?

Consider critical thinking first. We think critically when we have at least one problem to solve. One is not doing good critical thinking, therefore, if one is not solving any problems. If there is no problem there is no point in thinking critically. The "opposite" is also true. Uncritical problem solving is unintelligible. There is no way to solve problems effectively unless one thinks critically about the nature of the problems and of how to go about solving them. Thinking our way through a problem to a solution, then, is critical thinking, not something else. Furthermore, critical thinking, because it involves our working out afresh our own thinking on a subject, and because our own thinking is always a unique product of our self-structured experience, ideas, and reasoning, is intrinsically a new "creation", a new "making", a new set of cognitive and affective structures of some kind. All thinking, in short, is a creation of the mind's work, and when it is disciplined so as to be well-integrated into our experience, it is a new creation precisely because of the inevitable novelty of that integration. And when it helps us to solve problems that we could not solve before, it is surely properly called "creative".

How do communication skills fit in?

Some communication is surface communication, trivial communication--surface and trivial communication don't really require education. All of us can engage in small talk, can share gossip. And we don't require any intricate skills to do that fairly well. Where communication becomes part of our educational goal is in reading, writing, speaking and listening. These are the four modalities of communication which are essential to education and each of them is a mode of reasoning. Each of them involves problems. Each of them is shot through with critical thinking needs. Take the apparently simple matter of reading a book worth reading. The author has developed her thinking in the book, has taken some ideas and in some way represented those ideas in extended form. Our job as a reader is to translate the meaning of the author into meanings that we can understand.

And self esteem? How does it fit in?

Healthy self-esteem emerges from a justified sense of self-worth, just as self-worth emerges from competence, ability, and genuine success. If one simply feels good about oneself for no good reason, then one is either arrogant (which is surely not desirable) or, alternatively, has a dangerous sense of misplaced confidence. Teenagers, for example, sometimes think so well of themselves that they operate under the illusion that they can safely drive while drunk or safely take drugs. They often feel much too highly of their own competence and powers and are much too unaware of their limitations. To accurately sort out genuine self-worth from a false sense of self-esteem requires, yes you guessed it, critical thinking.

And finally, what about collaborative learning? How does it fit in?

Collaborative learning is desirable only if grounded in disciplined critical thinking. Without critical thinking, collaborative learning is likely to become collaborative mis-learning. It is collective bad thinking in which the bad thinking being shared becomes validated. Remember, gossip is a form of collaborative learning; peer group indoctrination is a form of collaborative learning; mass hysteria is a form of speed collaborative learning (mass learning of a most undesirable kind). We learn prejudices collaboratively, social hates and fears collaboratively, stereotypes and narrowness of mind, collaboratively. If we don’t put disciplined critical thinking into the heart and soul of the collaboration, we get the mode of collaboration which is antithetical to education, knowledge, and insight.

One important aim of schooling should be to create a climate that evokes children’s sense of wonder and inspires their imagination to soar. What can teachers do to "kindle" this spark and keep it alive in education?

First of all, we kill the child's curiosity, her desire to question deeply, by superficial didactic instruction. Young children continually ask why. Why this and why that? And why this other thing? But we soon shut that curiosity down with glib answers, answers to fend off rather than to respond to the logic of the question. In every field of knowledge, every answer generates more questions, so that the more we know the more we recognize we don't know. It is only people who have little knowledge who take their knowledge to be complete and entire. If we thought deeply about almost any of the answers which we glibly give to children, we would recognize that we don't really have a satisfactory answer to most of their questions. Many of our answers are no more than a repetition of what we as children heard from adults. We pass on the misconceptions of our parents and those of their parents. We say what we heard, not what we know. We rarely join the quest with our children. We rarely admit our ignorance, even to ourselves. Why does rain fall from the sky? Why is snow cold? What is electricity and how does it go through the wire? Why are people bad? Why does evil exist? Why is there war? Why did my dog have to die? Why do flowers bloom? Do we really have good answers to these questions?

How does curiosity fit in with critical thinking?

To flourish, curiosity must evolve into disciplined inquiry and reflection. Left to itself it will soar like a kite without a tail, that is, right into the ground! Intellectual curiosity is an important trait of mind, but it requires a family of other traits to fulfill it. It requires intellectual humility, intellectual courage, intellectual integrity, intellectual perseverance, and faith in reason. After all, intellectual curiosity is not a thing in itself — valuable in itself and for itself. It is valuable because it can lead to knowledge, understanding, and insight; because it can help broaden, deepen, sharpen our minds, making us better, more humane, more richly endowed persons.

It is important for our students to be productive members of the work-force. How can schools better prepare students to meet these challenges?

The fundamental characteristic of the world students now enter is ever-accelerating change; a world in which information is multiplying even as it is swiftly becoming obsolete and out of date; a world in which ideas are continually restructured, retested, and rethought; where one cannot survive with simply one way of thinking; where one must continually adapt one's thinking to the thinking of others; where one must respect the need for accuracy and precision and meticulousness; a world in which job skills must continually be upgraded and perfected — even transformed. We have never had to face such a world before. Education has never before had to prepare students for such dynamic flux, unpredictability, and complexity for such ferment, tumult, and disarray.

National standards will result in national accountability. What is your vision for the future?

Most of the national assessment we have done thus far is based on lower-order learning and thinking. It has focused on what might be called surface knowledge. It has rewarded the kind of thinking that lends itself to multiple choice machine-graded assessment. We now recognize that the assessment of the future must focus on higher – not lower – order thinking; that it must assess more reasoning than recall; that it must assess authentic performances, students engaged in bona fide intellectual work.

by Richard Paul.}

 
 
 

Have a language expert improve your writing

Run a free plagiarism check in 10 minutes, generate accurate citations for free.

  • Knowledge Base
  • Working with sources
  • What Is Critical Thinking? | Definition & Examples

What Is Critical Thinking? | Definition & Examples

Published on May 30, 2022 by Eoghan Ryan . Revised on May 31, 2023.

Critical thinking is the ability to effectively analyze information and form a judgment .

To think critically, you must be aware of your own biases and assumptions when encountering information, and apply consistent standards when evaluating sources .

Critical thinking skills help you to:

  • Identify credible sources
  • Evaluate and respond to arguments
  • Assess alternative viewpoints
  • Test hypotheses against relevant criteria

Table of contents

Why is critical thinking important, critical thinking examples, how to think critically, other interesting articles, frequently asked questions about critical thinking.

Critical thinking is important for making judgments about sources of information and forming your own arguments. It emphasizes a rational, objective, and self-aware approach that can help you to identify credible sources and strengthen your conclusions.

Critical thinking is important in all disciplines and throughout all stages of the research process . The types of evidence used in the sciences and in the humanities may differ, but critical thinking skills are relevant to both.

In academic writing , critical thinking can help you to determine whether a source:

  • Is free from research bias
  • Provides evidence to support its research findings
  • Considers alternative viewpoints

Outside of academia, critical thinking goes hand in hand with information literacy to help you form opinions rationally and engage independently and critically with popular media.

Scribbr Citation Checker New

The AI-powered Citation Checker helps you avoid common mistakes such as:

  • Missing commas and periods
  • Incorrect usage of “et al.”
  • Ampersands (&) in narrative citations
  • Missing reference entries

critical thinking assumption questions

Critical thinking can help you to identify reliable sources of information that you can cite in your research paper . It can also guide your own research methods and inform your own arguments.

Outside of academia, critical thinking can help you to be aware of both your own and others’ biases and assumptions.

Academic examples

However, when you compare the findings of the study with other current research, you determine that the results seem improbable. You analyze the paper again, consulting the sources it cites.

You notice that the research was funded by the pharmaceutical company that created the treatment. Because of this, you view its results skeptically and determine that more independent research is necessary to confirm or refute them. Example: Poor critical thinking in an academic context You’re researching a paper on the impact wireless technology has had on developing countries that previously did not have large-scale communications infrastructure. You read an article that seems to confirm your hypothesis: the impact is mainly positive. Rather than evaluating the research methodology, you accept the findings uncritically.

Nonacademic examples

However, you decide to compare this review article with consumer reviews on a different site. You find that these reviews are not as positive. Some customers have had problems installing the alarm, and some have noted that it activates for no apparent reason.

You revisit the original review article. You notice that the words “sponsored content” appear in small print under the article title. Based on this, you conclude that the review is advertising and is therefore not an unbiased source. Example: Poor critical thinking in a nonacademic context You support a candidate in an upcoming election. You visit an online news site affiliated with their political party and read an article that criticizes their opponent. The article claims that the opponent is inexperienced in politics. You accept this without evidence, because it fits your preconceptions about the opponent.

There is no single way to think critically. How you engage with information will depend on the type of source you’re using and the information you need.

However, you can engage with sources in a systematic and critical way by asking certain questions when you encounter information. Like the CRAAP test , these questions focus on the currency , relevance , authority , accuracy , and purpose of a source of information.

When encountering information, ask:

  • Who is the author? Are they an expert in their field?
  • What do they say? Is their argument clear? Can you summarize it?
  • When did they say this? Is the source current?
  • Where is the information published? Is it an academic article? Is it peer-reviewed ?
  • Why did the author publish it? What is their motivation?
  • How do they make their argument? Is it backed up by evidence? Does it rely on opinion, speculation, or appeals to emotion ? Do they address alternative arguments?

Critical thinking also involves being aware of your own biases, not only those of others. When you make an argument or draw your own conclusions, you can ask similar questions about your own writing:

  • Am I only considering evidence that supports my preconceptions?
  • Is my argument expressed clearly and backed up with credible sources?
  • Would I be convinced by this argument coming from someone else?

If you want to know more about ChatGPT, AI tools , citation , and plagiarism , make sure to check out some of our other articles with explanations and examples.

  • ChatGPT vs human editor
  • ChatGPT citations
  • Is ChatGPT trustworthy?
  • Using ChatGPT for your studies
  • What is ChatGPT?
  • Chicago style
  • Paraphrasing

 Plagiarism

  • Types of plagiarism
  • Self-plagiarism
  • Avoiding plagiarism
  • Academic integrity
  • Consequences of plagiarism
  • Common knowledge

Don't submit your assignments before you do this

The academic proofreading tool has been trained on 1000s of academic texts. Making it the most accurate and reliable proofreading tool for students. Free citation check included.

critical thinking assumption questions

Try for free

Critical thinking refers to the ability to evaluate information and to be aware of biases or assumptions, including your own.

Like information literacy , it involves evaluating arguments, identifying and solving problems in an objective and systematic way, and clearly communicating your ideas.

Critical thinking skills include the ability to:

You can assess information and arguments critically by asking certain questions about the source. You can use the CRAAP test , focusing on the currency , relevance , authority , accuracy , and purpose of a source of information.

Ask questions such as:

  • Who is the author? Are they an expert?
  • How do they make their argument? Is it backed up by evidence?

A credible source should pass the CRAAP test  and follow these guidelines:

  • The information should be up to date and current.
  • The author and publication should be a trusted authority on the subject you are researching.
  • The sources the author cited should be easy to find, clear, and unbiased.
  • For a web source, the URL and layout should signify that it is trustworthy.

Information literacy refers to a broad range of skills, including the ability to find, evaluate, and use sources of information effectively.

Being information literate means that you:

  • Know how to find credible sources
  • Use relevant sources to inform your research
  • Understand what constitutes plagiarism
  • Know how to cite your sources correctly

Confirmation bias is the tendency to search, interpret, and recall information in a way that aligns with our pre-existing values, opinions, or beliefs. It refers to the ability to recollect information best when it amplifies what we already believe. Relatedly, we tend to forget information that contradicts our opinions.

Although selective recall is a component of confirmation bias, it should not be confused with recall bias.

On the other hand, recall bias refers to the differences in the ability between study participants to recall past events when self-reporting is used. This difference in accuracy or completeness of recollection is not related to beliefs or opinions. Rather, recall bias relates to other factors, such as the length of the recall period, age, and the characteristics of the disease under investigation.

Cite this Scribbr article

If you want to cite this source, you can copy and paste the citation or click the “Cite this Scribbr article” button to automatically add the citation to our free Citation Generator.

Ryan, E. (2023, May 31). What Is Critical Thinking? | Definition & Examples. Scribbr. Retrieved August 19, 2024, from https://www.scribbr.com/working-with-sources/critical-thinking/

Is this article helpful?

Eoghan Ryan

Eoghan Ryan

Other students also liked, student guide: information literacy | meaning & examples, what are credible sources & how to spot them | examples, applying the craap test & evaluating sources, "i thought ai proofreading was useless but..".

I've been using Scribbr for years now and I know it's a service that won't disappoint. It does a good job spotting mistakes”

  • International
  • Education Jobs
  • Schools directory
  • Resources Education Jobs Schools directory News Search

“Glubbslyme” by Jacqueline Wilson: Novel Study Worksheets with Questions & Answers

“Glubbslyme” by Jacqueline Wilson: Novel Study Worksheets with Questions & Answers

Subject: English

Age range: 7-11

Resource type: Worksheet/Activity

SweetSuccess's Shop

Last updated

19 August 2024

  • Share through email
  • Share through twitter
  • Share through linkedin
  • Share through facebook
  • Share through pinterest

critical thinking assumption questions

These novel study printable pages for “Glubbslyme” by Jacqueline Wilson are designed to engage young readers with a mix of critical thinking, comprehension and fun question activities that bring the story to life. Perfect for classroom use or at-home learning, these worksheets will guide students through a deeper understanding of the novel while honing their reading and analytical skills.

Our novel study package includes:

- Multiple Choice: Test students’ recall and understanding of key plot points, character relationships and important details with a series of carefully crafted multiple-choice questions.

- Comprehension Questions: Encourage deeper thinking and interpretation with open-ended questions that require full-sentence answers, helping students to express their thoughts and reflections on the book.

- Answer Booklet: A complete answer key is included, making it easy for educators and parents to assess student progress and understanding quickly and efficiently.

Ideal for ages 7-13, these printable worksheets are an excellent resource for fostering a love of reading and literature. They are designed to challenge students appropriately while ensuring that they remain engaged and excited about their learning journey. Whether you are a teacher seeking classroom resources or a parent wanting to support your child’s education, these are a valuable tool for enhancing the reading experience of “Glubbslyme.” Buy these printable pages today and watch your students’ comprehension and appreciation for this classic tale grow!

CONTAINS: 15 PAGES

Thanks a million for visiting Sweet Success Supplies! May your endeavors be as sweet as our deals!

Tes paid licence How can I reuse this?

Your rating is required to reflect your happiness.

It's good to leave some feedback.

Something went wrong, please try again later.

This resource hasn't been reviewed yet

To ensure quality for our reviews, only customers who have purchased this resource can review it

Report this resource to let us know if it violates our terms and conditions. Our customer service team will review your report and will be in touch.

Not quite what you were looking for? Search by keyword to find the right resource:

IMAGES

  1. Critical Thinking Questions / Blooms Taxonomy / Teacher Printable

    critical thinking assumption questions

  2. PPT

    critical thinking assumption questions

  3. PPT

    critical thinking assumption questions

  4. Critical Thinking Worksheet: Facts and Assumptions in 2023

    critical thinking assumption questions

  5. 6 Critical Thinking Questions For Any Situation

    critical thinking assumption questions

  6. Critical Thinking in the Classroom

    critical thinking assumption questions

COMMENTS

  1. How to Tackle Critical Reading Assumption Questions

    Luckily, arguments on GMAT Critical Reasoning questions are relatively formulaic, so let's go over the basics first: A premise is the starting point of the argument. The conclusion is what the author wants you to believe by the end of the argument. The assumption is the missing link between the premise and conclusion.

  2. Identify Assumptions: Steps to Question Underlying Beliefs in Decision

    Assumptions are the unspoken bedrock of beliefs and propositions that shape the way individuals perceive the world and inform their decision-making process. These underpinnings, often accepted as truth, play a critical role in forming perspective and dictate the importance placed on certain topics or actions.

  3. Critical Thinking and Academic Research: Assumptions

    Question Assumptions. An assumption is an unexamined belief: what we think without realizing we think it. Our inferences (also called conclusions) are often based on assumptions that we haven't thought about critically. A critical thinker, however, is attentive to these assumptions because they are sometimes incorrect or misguided.

  4. How to Tackle Critical Reading Assumption Questions

    Assumption questions ask you to find the unstated link between a question's premise and its conclusion. Assumptions are crucial in understanding and refuting arguments, so they play a large role in two major Critical Reasoning question types.In this post, we'll cover GMAT Critical Reasoning tips and practice questions to help you tackle assumption questions.

  5. GRE Critical Reasoning Question Type: Assumption Questions

    If you negate an answer choice and it does not do anything to the conclusion, then you know that answer choice is not an assumption the argument depends on. Look at the answers below: Studying for three months automatically means a student will score in the top 10% on the GRE. Using prep materials is the only way to score in the top 10%.

  6. What About Assumptions?

    Assumptions are beliefs or ideas that are believed to be true without proof or evidence and are used to support reasoning. This lack of verification can create bias when thinking critically. Like any human activity, the practice of critical thinking requires several basic assumptions to make sense. For people who don't share these assumptions ...

  7. Critical Thinking: Challenging Assumptions

    CPE. 4.6 (1395) From Channel: Critical Thinking. Thinking Critically. Every day, we make hundreds of assumptions based on facts, observations, and experiences. Some are useful time savers, but some are based on a weak premise or poor data and can lead to jumping to a conclusion without asking questions or listening to other people's input.

  8. GMAT Critical Reasoning

    Another common Critical Reasoning tip you will see across most test prep curricula is the assumption-negation technique. In short, this technique can be used to deal with difficult negation in answer choices for assumption questions. If you negate an essential premise (which is the correct answer in an assumption question) then it should ...

  9. Critical Reasoning Assumption Questions

    Step 3: Pause and State the Goal (Brainstorm / Try to Predict the Right Answer) For any Critical Reasoning questions that are part of what we call the "assumption family"—assumption, strengthen, weaken, or evaluate—you always want to try to kick the tires on the argument before looking at the answer choices.

  10. Introduction to GMAT Critical Reasoning

    The questions revolve around a logical analysis of the core argument and might involve strengthening it, weakening it, finding its underlying assumption, etc. GMAT Critical Reasoning tests your critical thinking and logic skills more than your reading skills. Logical reasoning is also one of the most important skills in the business world.

  11. Critical Reasoning: Key Concepts, Types, Tricks, Sample Questions

    In these critical thinking reasoning questions, by reading the given statements, candidates need to take the right decision. Here taking the right decision means selecting the correct assumption. 3. Statement & Conclusion . In these type of critical reasoning questions, passage or statements will be given followed by some conclusions.

  12. Assumptions Tutorial

    In this Assumptions Tutorial, our expert test developer talks through how to answer an assumptions question from a critical thinking test.In these questions ...

  13. PDF Distinguishing Between Inferences and Assumptions

    To be skilled in critical thinking is to be able to take one's thinking apart systematically, to analyze each part, assess it for quality and then improve it. The first step in this process is understanding the parts of thinking, or elements of reasoning. These elements are: purpose, question, information, inference, assumption, point of

  14. Questioning Assumptions: A Critical Thinking Skill

    Questioning assumptions is a powerful tool for problem solving and critical thinking. It is a way to uncover hidden assumptions and biases, and look at situations from different angles. By actively questioning our beliefs, thoughts, and ideas, we can develop better solutions, make better decisions, and create more effective outcomes.

  15. What About Assumptions?

    Here is a partial list of assumptions that sometimes cause trouble for people new to critical thinking. Critical thinking (CT) is evaluative. An evaluation is a statement that compares what is the case to a standard about how things should be. CT requires people to make lots of judgments about good and bad, right and wrong, what we should or ...

  16. Avoid Making Assumptions

    Overall, avoiding assumptions in the workplace promotes critical thinking by fostering objectivity, open-mindedness, better decision-making, enhanced problem-solving abilities, and effective communication. These skills are valuable for individual growth, team collaboration, and overall organizational success.

  17. Watson-Glaser Assessments: Guidelines & Practice Examples

    The Watson Glaser Assessment (Watson Glaser Critical Thinking Appraisal) is designed by Pearson Talentlens - a part of the famous Pearson education publishing house. The assessment is a quick, consistent, and accurate measurement of the test-takers ability to analyze, reason, interpret and draw logical conclusions from written information.

  18. Free Critical Thinking Test: Sample Questions & Explanations

    The Five Critical Thinking Skills Explained. 1. Recognition of Assumption. You'll be presented with a statement. The statement is then followed by several proposed assumptions. When answering, you must work out if an assumption was made or if an assumption was not made in the statement.

  19. Distinguishing Between Inferences and Assumptions

    Distinguishing Between Inferences and Assumptions. To be skilled in critical thinking is to be able to take one's thinking apart systematically, to analyze each part, assess it for quality and then improve it. The first step in this process is understanding the parts of thinking, or elements of reasoning. These elements are: purpose, question ...

  20. 5 Questions to Improve Your Critical Thinking Skills [Part 1]

    Assumption 1: Since the past models of this car have functioned well on multiple occasions, the new model will function just as well. Assumption 2: The way that you will be driving this particular ...

  21. Critical Thinking: Basic Questions & Answers

    Two things are crucial: 1) critical thinking is not just thinking, but thinking which entails self-improvement. 2) this improvement comes from skill in using standards by which one appropriately assesses thinking. To put it briefly, it is self-improvement (in thinking) through standards (that assess thinking).

  22. What Is Critical Thinking?

    Critical thinking is the ability to effectively analyze information and form a judgment. To think critically, you must be aware of your own biases and assumptions when encountering information, and apply consistent standards when evaluating sources. Critical thinking skills help you to: Identify credible sources. Evaluate and respond to arguments.

  23. Critical thinking

    Critical thinking is the analysis of available facts, evidence, observations, and arguments in order to form a judgement by the application of rational, skeptical, and unbiased analyses and evaluation. [1] In modern times, the use of the phrase critical thinking can be traced to John Dewey, who used the phrase reflective thinking. [2] The application of critical thinking includes self-directed ...

  24. "Glubbslyme" by Jacqueline Wilson: Novel Study Worksheets ...

    - Multiple Choice: Test students' recall and understanding of key plot points, character relationships and important details with a series of carefully crafted multiple-choice questions. - Comprehension Questions: Encourage deeper thinking and interpretation with open-ended questions that require full-sentence answers, helping students to ...